TỔ HỢP TỔNG HỢP

You might also like

Download as pdf or txt
Download as pdf or txt
You are on page 1of 60

MỘT SỐ BÀI TOÁN TỔ HỢP SUY LUẬN TỔNG HỢP

1. Cơ sở lý thuyết cơ bản
Không có một phương pháp chung nào cho việc giải các bài toán suy luận tổng
hợp. Tùy thuộc vào từng bài toán mà người giải vận dụng linh hoạt các nguyên lý
như: nguyên lý phản chứng, nguyên lý quy nạp, nguyên lý Dirichlet,… hoặc đưa ra
các nhận xét tinh tế để từ đó hình thành lời giải phù hợp. Với các bài toán suy luận ta
thường phải phối hợp các kiến thức một cách hợp lý, muốn vậy người giải ngoài việc
phải nắm kỹ các kiến thức toán mà còn phải rèn luyện khả năng phân tích, nhận xét,
dự đoán các kết quả có thể xảy ra.
2. Một số ví dụ minh họa
Ví dụ 1. Cho tập hợp A  1; 2; 3;...; 2006 2007  . Chứng minh rằng trong 2007 số khác
2007
nhau tùy ý được lập từ tập hợp trên có ít nhất hai số x, y thỏa mãn 0  x  2007 y  1 .

Lời giải
Gọi 2007 số đã cho là 1  a1  a 2  a 3  ...  a 2007  20062007 . Xét 2006 số sau:

b1  2007 a 2  2007 a1 ; b 2  2007 a 3  2007 a 2 ; ...; b 2006  2007 a 2007  2007 a 2006

Ta có b1 , b2 ,...; b2006  0

Giả sử không có số bi  i  1, 2,..., 2006  nào nhỏ hơn 1, suy ra b1 , b2 ,..., b2006  1 , do đó

b1  b2  ...  b2006  1  1  ...  1  2006

Mặt khác ta lại có


b1  b2  ...  b 2006  2007 a 2  2007 a1  2007 a 3  2007 a 2  ...  2007 a 2007  2007 a 2006
 2007 a 2007  2007 a1

Từ đó ta suy ra được 2007 a 2007  2007 a 1  2006  2007 a 2007  2006  2007 a 1  2006

Hay a 2007  2006 2007 , điều này trái với giả thiết nên giả sử là sai. Vậy ta có điều phải
chứng minh.
Ví dụ 2. Cho tứ giác lồi ABCD có độ dài bốn cạnh là a, b, c, d đều là các số nguyên
dương. Chứng minh rằng nếu độ dài mỗi cạnh đều là ước số của chu vi tứ giác này thì
tứ giác đó có ít nhất hai cạnh bằng nhau.

1
Lời giải
Giả sử có tứ giác thỏa mãn yêu cầu đề bài mà không tồn tại hai cạnh nào của tứ giác
bằng nhau. Khi đó không mất tính tổng quát, ta sắp thứ tự a  b  c  d  1 .
a  b  c  d  ma

a  b  c  d  nb
Vì độ dài mỗi cạnh đều là ước số của chu vi tứ giác nên 
a  b  c  d  pc
a  b  c  d  qd

Với m, n, p,q  N* và m  n  p  q

1 a
m  a  b  c  d

1  b
 n a  b  c  d 1 1 1 1
Suy ra ra được  1 c
    1
  m n p q
p a  b  c  d
1 d
 
 q a  b  c  d

Do đó ABCD là tứ giác lồi nên a  b  c  d  3a nên 2a  a  b  c  d  4a .


a  bcd1  1 1 1 1 1 1 1 1
Mặt khác ta có   m  3 và        1
a  b  c  d  am  m n p q 3 4 5 6

Ta có mâu thuẫn nên điều giả sử là sai.


Vậy bài toán được chứng minh.
1 1 1
Ví dụ 3. Cho 25 số tự nhiên a1 ; a 2 ;...; a 25 khác 0 thoả mãn     9.
a1 a2 a 25
Chứng minh rằng trong 25 số tự nhiên đó luôn tồn tại hai số bằng nhau.
Lời giải
Giả sử trong 25 số tự nhiên a1 ; a 2 ; ...; a 25 không có hai số nào bằng nhau.
Không mất tính tổng quát ta có thể chọn a1  a 2  ...  a 25 . Khi đó ta có
a1  1; a 2  2;...; a 25  25

1 1 1 1 1 1
Suy ra ta được         
a1 a2 a 25 1 2 25

Mặt khác ta chứng minh được

2
1 1 1 2 2 2
     1   ... 
1 2 25 2 2 2 3 2 25
 1 1 1 
 1 2   ...  
 2 1 3 2 25  24 
 1 2 2  1  3  2  ......  25  24 
 1 2 25  1  9 
1 1 1
Điều này dẫn tới     9
a1 a2 a 25

Bất đẳng thức thu được mâu thuẫn với giả thiết của bài toán.
Vậy điều ta giả sử là không xảy ra hay bài toán được chứng minh.
Ví dụ 4. Cho 2015 số tự nhiên a1 ; a 2 ;...; a 2015 khác 0 thoả mãn điều kiện:
1 1 1 1
   ...   89
a1 a2 a3 a 2015
Chứng minh rằng trong 2015 số tự nhiên đó luôn tồn tại hai số bằng nhau.

Lời giải
Giả sử trong 2015 số tự nhiên a1 ; a 2 ;...; a 2015 không có hai số nào bằng nhau. Không
mất tính tổng quát ta có thể chọn a1  a 2  ...  a 2015 . Khi đó ta có
a1  1; a 2  2; a 3  3;...; a 2015  2015
1 1 1 1 1 1 1 1
Suy ra ta được    ...      ... 
a1 a2 a3 a 2015 1 2 3 2015
Mặt khác ta chứng minh được
1 1 1 2 2 2
     1   ... 
1 2 2015 2 2 2 3 2 2015
 1 1 1 
 1 2   ...  
 2 1 3 2 2015  2014 
 1 2 2  1  3  2  ...  2015  2014 
 1 2 2015  1  89 
1 1 1
Điều này dẫn tới      89 .
a1 a2 a 2015
Bất đẳng thức thu được mâu thuẫn với giả thiết của bài toán.
Vậy điều ta giả sử là không xảy ra hay bài toán được chứng minh.

3
Ví dụ 5. Với mỗi số nguyên dương được gán một trong ba màu: xanh, đỏ, vàng.
Chứng minh rằng luôn tồn tại 2 số nguyên dương phân biệt x, y sao cho x, y giống
màu nhau thỏa mãn x  y là một số chính phương.

Lời giải
Giải sử không tồn tại hai số x, y nào thỏa mãn yêu cầu bài toán, tức là nếu x  y là

một số chính phương thì x và y khác màu nhau.


Nhận xét:
+ Với a  10 và a là số nguyên dương thì bộ ba  a; a  9; a  16  có tính chất trị tuyệt đối

hiệu của bất kì hai phần tử nào trong bộ là một số chính phương.
Theo giả thiết phản chứng, ta có  a; a  9; a  16  là ba số đôi một khác màu nhau. Mặt

khác, bộ ba  a  7; a  9; a  16  cũng có tính chất như bộ ba ba đầu do đó theo giả thiết

phản chứng, ta có  a  7; a  9; a  16  là ba số đôi một khác màu nhau.

Suy ra: a và a + 7 cùng màu.


+ Tăng mỗi số ở bộ ba ban đầu lên 7 đơn vị ta được bộ ba mới, sau đó tăng số đầu ở
bộ ba mới này thêm 7 đơn vị, ta cũng chỉ ra được: a + 7 và a + 14 cùng màu.
+ Tiếp tục quá trình trên ta được các số a; a  7; a  14; a  21; ...; a  49 cùng màu. Mặt

khác a  49  a  49 là một số chính phương.

Suy ra mâu thuẫn. Vậy giả sử ban đầu sai, do đó luôn luôn tồn tại 2 số x, y sao cho
x  y là một số chính phương và x, y cùng màu. Đến đây ta có điều phải chứng minh.

Ví dụ 6. Giả sử a1 ,a 2 ,...,a11 là các số nguyên dương lớn hơn bằng 2 và đôi một khác
nhau thỏa mãn điều kiện a1  a 2  ...  a11  407. Tồn tại hay không số nguyên dương n
sao cho tổng các số dư của các phép chia n cho 22 số a1 ,a 2 ,...,a11 , 4a1 , 4a 2 ,..., 4a11 bằng
2012.
Lời giải
Giả sử tồn tại số n thỏa mãn yêu cầu bài toán, khi đó tổng số dư khi chia n cho
a 1 ,a 2 ,...,a 11 không vướt quá 407 – 11  396 . Tổng số dư khi chia n cho 4a 1 , 4a 2 ,..., 4a 11

không vướt quá 4.407 – 11  1617 .

4
Do đó tổng số dư là 2013, điều này chứng tỏ có 21 số dư lớn nhất và 1 số dư nhỏ hơn
số chia 2 đơn vị.
Giả sử tồn tại k để a k ; 4a k thỏa mãn 1 số dư nhỏ hơn số chia 2 đơn vị, khi đó
 (n  1) a k

 (n  2) (4a k )  (n  1) a k  1 a
 (n  1) (4a ) 
(n  2) a k
k
 k

 (n  2) a k

Điều này không đúng do ak ≥ 2 theo giả thiết.


Vậy không tồn tại số n thỏa mãn yêu cầu bài toán.
Ví dụ 7. a) Tìm các số nguyên a, b, c sao cho a  b  c  0 và ab  bc  ca  3  0
b) Cho m là số nguyên, chứng minh rằng nếu tồn tại các số nguyên a, b, c
khác 0 sao cho a  b  c  0 và ab  bc  ca  4m  0 thì cũng tồn tại các số nguyên a', b',
c' khác 0 sao cho a '  b'  c'  0 và a ' b' b' c ' c 'a ' m  0 .
c) Với k là số nguyên dương, chứng minh rằng không tồn tại các số nguyên
a, b, c khác 0 sao cho a  b  c  0 và ab  bc  ca  2 k  0 .
Lời giải
a  b  c  0 a 2  b2  c 2  2  ab  bc  ca   0
a) Ta có:    a 2  b2  c 2  6
ab  bc  ca  3  0 ab  bc  ca  3

Vì a, b,c  Z và a 2 , b2 ,c 2 là các số chính phương nên ta được a 2 , b2 , c 2  0; 1; 4 do đó

ta được a, b,c  0;  1;  2 mà a  b  c  0 . Từ đó ta chọn được các bộ số như sau

a  2; b  c  1 hoặc a  2; b  c  1 và các hoán vị của chúng.

b) Nếu a  b  c  0 thì trong ba số a, b, c có hai số lẻ và một số chẵn hoặc cả ba số


cùng chẵn.
Nếu có hai số lẻ và một số chẵn, không mất tính tổng quát ta giả sử a, b lẻ và c chẵn
Khi đó ta được a  2a ' 1; b  2b' 1; c  2c'  a '; b'; c'  Z  .

Suy ra
2 2 2
 
8m  a 2  b 2  c 2   2a ' 1   2b' 1   2c'   4 a '2  b'2  c'2  a ' b'  2

Dễ thấy 8m chia hết cho 4 và 4  a '2  b'2  c '2  a ' b' m   2 không chia hết cho 4. Điều

này dẫn đến mâu thuẫn. Vậy trường hợp này không xảy ra.

5
a b c
Do đó cả ba số a, b, c cúng chẵn, ta chọn a '  ; b'  ; c '  suy ra a '  b'  c'  0
2 2 2
ab  bc  ca 4m
Và ta cũng có a ' b'  b' c'  c'a '  m  m  m 0
4 4
Như vậy các số a '; b'; c ' thỏa mãn yêu cầu bài toán.
c) Nếu a  b  c  0 và ab  bc  ca  2k  0 thì tương tự câu a ta có
a 2  b 2  c 2  2.2 k  2 k 1
Do a, b, c là các số nguyên khác 0, có tổng bằng 0 nên tồn tại 2 số cùng dấu và trị
tuyệt đối số còn lại bằng tổng các trị tuyệt đối của 2 số đó, nghĩa là trị tuyệt đối số
còn lại ≥2 nên a 2  b2  c 2  6 nên k  2 , do đó 2 k  4
Vì a  b  c  0 nên trong ba số a, b, c có hai số lẻ và một số chẵn hoặc cả ba số cùng
chẵn.
+ Trường hợp 1: Giả sử trong ba số a, b, c thì a, b là số và c là số chẵn. khi đó
a  2a ' 1; b  2b' 1; c  2c'  a '; b'; c'  Z 
2 2 2
Ta có 2 k 1  a 2  b2  c 2   2a ' 1   2b' 1   2c'   4  a '2  b'2  c '2  a ' b'   2

Dễ thấy 0 chia hết cho 4 và 4  a '2  b'2  c'2  a ' b'   2 không chia hết cho 4. Điều này

dẫn đến mâu thuẫn. Vậy trường hợp này không xẩy ra.
+ Trường hợp 2: cả ba số a, b, c cùng chẵn. Khi đó gọi p là số tự nhiên lớn nhất sao
cho cả ba số a, b, c cùng chia hết cho 2 p . Nghĩa là a  a '.2 p ; b  b'.2 p ; c  c'.2 p với
a’, b’, c’ là các số nguyên và có ít nhất một số lẻ.

Khi đó ta được a 2  b2  c 2  2 k 1   a '2  b'2  c'2  2 2p  2 k  a '2  b'2  c '2  2 k 2p

Tương tự như trên ta được a’, b’, c’ khác 0 nên a '2  b'2  c '2  6 nên k  2p  2 nên

2 k  2p  4 . Mà a’, b’, c’ khác 0 trong đó có ít nhất một số lẻ nên a '2  b'2  c '2 không chia

hết cho 4 (đem chia 4 dư 1 hoặc 2 hoặc 3). Do đó trường hợp này cũng không xảy ra.
Vậy không tồn tại các số nguyên a, b, c khác 0 thỏa mãn yêu cầu bài toán.
Ví dụ 8. Cho n là một số nguyên dương và các số nguyên dương a1 ; a 2 ; a 3 ;...; a n có
tổng bằng 2n  1 . Chứng minh rằng tồn tại một số số trong các số nguyên dương trên
có tổng bằng n.

6
Lời giải
Đặt b1  a1 ; b2  a1  a 2 ;...; bn  a1  a 2  ...  a n .
Vì a1 ; a 2 ; a 3 ;...; a n là các số nguyên dương nên ta được 1  b1  b2  ...  bn  2n  1 .
Ta xét các trường hợp sau:
+ Nếu n số nguyên dương b1 ; b2 ;...; bn khi chia cho n có các số dư khác nhau thì tồn
tại một số chia hết cho n. Không mất tính tổng quát, ta giả sử số đó là bk với

k  1; 2; 3;...; n

Khi đó ta có 1  bk  2n  1 nên ta được bk  n . Do đó ta được a1  a 2  a 3  ...  a k  n .


+ Nếu n số nguyên dương b1 ; b2 ;...; bn khi chia cho n có hai số dư bằng nhau. Không

mất tính tổng quát ta giả sử số đó là bk và bl với k, m  1; 2; 3; ...; n và k  m

Từ đó ta được bk  bm  n do hiệu này lớn hơn 0 và nhỏ hơn 2n-1 nên bk  b m  n hay

ta được  a1  a 2  a 3  ...  a k    a1  a 2  a 3  ...  a m   n

Từ đó ta được a l1  a l 2  ...  a k  n .


Vậy luôn tồn tại một số số trong các số nguyên dương trên có tổng bằng n.
Ví dụ 9. Chọn 100 số tự nhiên khác nhau bất kì sao cho mỗi số đều không vượt qua
2015 và mỗi số đều chia 17 dư 10. Chứng minh rằng trong 100 số trên luôn chọn
được ba số có tổng không lớn hơn 999.
Lời giải
Giả sử n là số tự nhiên chia 17 dư 10, khi đó n  0 và n có dạng n  17k  10 với
kN.
Gọi 100 số tự nhiên được chọn là 17k1  10;17k 2  10;17k 3  10;...;17k100  10 .
Không mất tính tổng quát ta giả sử k1  k 2  k 3  ...  k100 .
Nếu k100  118 thi khi đó 17k100  10  17.118  10  2016 . Do đó k100  117 .
Ta sẽ chứng minh k 3  20 . Thật vậy, giả sử k 3  21 .
Khi đó từ k1  k 2  k 3  ...  k100 suy ra k 4  k 3  1; k5  k4  1; k6  k 5  1;...; k100  k99  1
Nên từ k 3  21 suy ra k 4  21  1  22; k5  22  1  23; k6  23  1  24;...; k100  117  1  118 ,
điều này trái với k100  118 . Do đó k 3  20 .
Vì k 3  20 nên suy ra k 2  19; k1  18 .
Với kết quả trên ta chọn ba số nhỏ nhất trong 100 số trên là
17k1  10;17k 2  10;17k 3  10

7
Khi đó 17k1  10  17k 2  10  17k 3  10   17.18  10   17.19  10    17.20  10   999
Vậy ta luôn chọn được ba số có tổng không lớn hơn 999.
Bài toán được chứng minh.
Ví dụ 10. Cho một lớp học có 35 học sinh, các học sinh này tổ chức một số câu lạc
bộ môn học. Mỗi học sinh tham gia đúng một câu lạc bộ. Nếu chọn ra 10 học sinh bất
kì thì luôn có ít nhất 3 học sinh tham gia cùng một câu lạc bộ. Chứng minh có một
câu lạc bộ gồm ít nhất 9 học sinh.

Lời giải
Giả sử tất cả các câu lạc bộ đều có không quá 8 học sinh. Gọi n là số câu lạc bộ có
hơn 1 học sinh.
+ Nếu n  4 , khi đó từ 5 trong số các câu lạc bộ này, chọn mỗi câu lạc bộ 2 học sinh,
khi đó 10 học sinh này không thỏa mãn điều kiện bài toán.
+ Nếu n  4 , khi đó số học sinh tham gia các câu lạc bộ này không quá 3.8  24 , nghĩa
là còn ít nhất 35  24  11 học sinh, mỗi học sinh tham gia 1 câu lạc bộ mà câu lạc bộ
này chỉ có 1 học sinh. Chọn 10 học sinh trong số này, không thỏa mãn điều kiện bài
toán.
Vậy n  4 .
Số học sinh tham gia 4 câu lạc bộ này không quá 4.8  32 , nghĩa là còn ít nhất 3 học
sinh, mỗi học sinh tham gia 1 câu lạc bộ mà câu lạc bộ này chỉ có 1 học sinh. Chọn 2
trong số học sinh này và mỗi câu lạc bộ trên chọn 2 học sinh, khi đó 10 học sinh
không thỏa mãn điều kiện.
Vậy điều giả sử sai, nghĩa là tồn tại một câu lạc bộ có ít nhất 9 học sinh tham gia.

Ví dụ 11. Trong một hộp có chứa 2011 viên bi màu (mỗi viên bi có đúng 1 màu),
trong đó có 655 viên bi màu đỏ, 655 viên bi màu xanh, 656 viên bi màu tím và 45
viên bi còn lại là viên bi màu vàng hoặc màu trắng (mỗi màu ít nhất 1 viên). Người
ta lấy ra từ hộp 178 viên bi bất kì. Chứng minh rằng trong số các viên bi lấy ra luôn
có ít nhất 45 viên bi cùng màu. Nếu người ta chỉ lấy ra 177 viên bi bất kì thì kết quả
bài toán còn đúng không?
Lời giải
Gọi x, y, z lần lượt là số màu đỏ, màu xanh, màu tím trong một lần lấy. Số bi có màu
khác với ba màu trên nhỏ hơn hoặc bằng 45. Vì vậy x  y  z  178  45  133 , vì

8
x, y, z  N , nếu x  45, y  45, z  45 thì x  y  z  132  133 . Điều này mâu thuẫn với

x  y  z  133 . Vậy trong ba số x, y, z có ít nhất một số lớn hơn hoặc bằng 45. Bài

toán được chứng minh.


Nếu lấy số bi là 177 viên thì kết luận trên có thể sai. Chẳng hạn x  y  z  44
và 45 viên bi màu khác thì tổng số viên bi lấy được là 177 viên. Nhưng vì có 45 viên
bi gồm màu trắng và màu vàng nên số bi mỗi màu lớn hơn hoặc bằng 1, do đó không
thể lấy quá 44 viên cùng màu (trắng hoặc vàng) trong lần lấy này.
Ví dụ 12. Cho tập hợp A gồm 70 số nguyên dương không vượt quá 90. Gọi B là tập
hợp gồm các số có dạng x  y với x, y  A và x, y không nhất thiết phân biệt.
1) Chứng minh 68  B
2) Chứng minh B chứa 91 số nguyên liên tiếp
Lời giải
Do tập hợp A chứa 70 số nguyên dương không vượt quá 90 nên gọi C là tập hợp chứa
20 số nguyên dương không vượt quá 90 và các số thuộc tập hợp C thì không thuộc
tập hợp A ( C  A   )
1) Chứng minh 68  B
Xét 34 cặp số 1; 67  ,  2; 66  , ... ,  34; 34  , các số trong các cặp này đều thuộc A hoặc C
Do có 34 cặp mà C có 20 phần tử nên tồn tại 1 cặp không có phần tử nào của C tức là
có ít nhất 1 cặp số có các số thuộc tập hợp A. Từ đó ta có: 68  B.
2) Chứng minh B chứa 91 số nguyên liên tiếp
Ta sẽ chứng minh các số nguyên dương n thỏa mãn 42  n  140 đều thuộc tập hợp B.
* Với 42  n  90
- Nếu n là số lẻ:
n 1 n 1 n  1 
Xét cặp số 1; n –1 ,  2; n – 2  , ... ;  ;  , các số trong các cặp này đều thuộc
2  2 2 
n  1 43  1
A hoặc C. Do có   21 cặp mà C có 20 phần tử nên tồn tại 1 cặp không có
2 2
phần tử nào của C tức là có ít nhất 1 cặp số có các số thuộc tập hợp A. Từ đó ta có:
n  B.
- Nếu n là số chẵn:
n n n
Xét cặp số 1; n –1 ,  2; n – 2  , ... ;  ;  , các số trong các cặp này đều thuộc A hoặc
2 2 2
C.
n 42
Do có   21 cặp mà C có 20 phần tử nên tồn tại 1 cặp không có phần tử nào của
2 2
C tức là có ít nhất 1 cặp số có các số thuộc tập hợp A. Từ đó ta có: n  B.
* Với 91  n  140
- Nếu n là số lẻ:

9
n 1 n  1 n 1 
Xét (91  ) cặp số:  90; n – 90  ,  89; n – 89  , ... ;  ;  , các số trong các cặp này
2  2 2 
n 1 139  1
đều thuộc A hoặc C. Do có 91   91   21 cặp mà C có 20 phần tử nên tồn tại
2 2
1 cặp không có phần tử nào của C tức là có ít nhất 1 cặp số có các số thuộc tập hợp A. Từ đó
ta có: n  B.
- Nếu n là số chẵn:
n n n
Xét (91  ) cặp số  90; n – 90  ,  89; n – 89  , ... ;  ;  , các số trong các cặp này đều
2  2 2

n 140
thuộc A hoặc C. Do có 91   91   21 cặp mà C có 20 phần tử nên tồn tại 1 cặp
2 2
không có phần tử nào của C tức là có ít nhất 1 cặp số có các số thuộc tập hợp A. Từ
đó ta có: n  B.
Bài toán được chứng minh.

Ví dụ 13. Có bao nhiêu tập hợp con A của tập hợp 1; 2; 3; 4;...; 2014 thỏa mãn điều
y2
kiện A có ít nhất hai phần tử và nếu x  A, y  A, x  y thì A .
xy
Lời giải
Ta chứng minh có tập con A thỏa mãn yêu cầu bài toán khi và chỉ khi nó có dạng
y; 2y .
Thật vậy giả sử x  y là hai phần tử của A.
y2 y2
+ Nếu x  2y thì theo giả thiết z   A và z   y , do đó ta được x  y  z
xy xy
thuộc A và x  2y  2z . Áp dụng giả thiết cho cặp x; z thuộc A, ta được z1  A và
x  z  z1 , và cứ như thế ta suy ra A có vô số phần tử, điều này mâu thuẫn.
Vậy x  2y và đặc biệt là y  z .
Nếu x  2y khi đó y  z , ta được z  2y (do 2 phần tử bất kỳ thuộc A thì đều phần tử
y2 3
lớn sẽ ≤ hai lần phần tử bé). Từ đó ta nhận được z   2y hay x  y do đó
xy 2
z  x . Vậy ta được y  z  x , do đó cặp y; z thuộc A. Áp dụng giả thiết cho cặp y; z ,
ta được z1  A và x  z1  y , và cứ như thế ta suy ra A có vô số phần tử, điều này mâu
thuẫn.
Như vậy x  2y , do đó A có dạng y; 2y với y  1; 2; 3; 4; ...; 2014 . Số tập con A có
dạng y; 2y với y  1; 2; 3; 4; ...; 2014 là 1007.
Thử lại ta thấy các tập con A thỏa mãn yêu cầu bài toán.

10
Ví dụ 14. Trong một kỳ thi, 60 thí sinh phải giải 3 bài toán. Khi kết thúc kỳ thi, người
ta nhận thấy rằng: với hai thí sinh bất kỳ luôn có ít nhất một bài toán mà cả hai thí
sinh đó đều giải được. Chứng minh rằng:
a) Nếu có một bài toán mà mọi thí sinh đều không giải được thì phải có một bài
toán khác mà mọi thí sinh đều giải được.
b) Có một bài toán mà có ít nhất 40 thí sinh giải được.
Lời giải
Gọi ba bài toán là A, B, C.
a) Không mất tính tổng quát, giả sử mọi thí sinh đều không giải được bài toán A.
 Nếu mọi thí sinh đều không giải được bài toán B thì từ giả thiết ta có mọi thí sinh

đều giải được bài toán C.


 Nếu mọi thí sinh đều giải được bài toán B và bài toán C thì ta có mọi thí sinh đều
giải được bài toán B; bài toán C.
 Nếu có một thí sinh chỉ giải được một bài toán, giả sử giải được bài toán B. Xét

học sinh này với tất cả các học sinh còn lại. Theo giả thiết, có mọi thí sinh đều giải
được bài toán B.
Vậy nếu có một bài toán mà mọi thí sinh đều không giải được thì phải có một bài
toán khác mà mọi thí sinh đều giải được.
b) Theo giả thiết ta có mọi thí sinh đều giải được ít nhất một bài toán. Nếu có một thí
sinh chỉ giải đúng một bài toán. Xét học sinh này với tất cả các học sinh còn lại, ta có
mọi thí sinh đều giải được bài toán đó. Ta chỉ còn xét trường hợp mà mọi thí sinh giải
được ít nhất hai bài toán.
Gọi số thí sinh giải được A, B mà không giải được C là x, số thí sinh giải
được B, C mà không giải được A là y, số thí sinh giải được A, C mà không giải được
B là z, số thí sinh giải được cả A, B, C là t (x, y, z, t  N) . Ta có x  y  z  t  60 (1)
Cách 1: Giả sử có điều trái với kết luận của bài toán.
Ta có x  z  t  40; x  y  t  40; y  z  t  40
Do đó x  z  t  x  y  t  y  z  t  40  40  40  2  x  y  z  t   t  120 .

Kết hợp (1) ta có t  0 . Điều này vô lí. Điều giả sử ở trên là sai.
Vậy có một bài toán mà có ít nhất 40 thí sinh giải được.

11
Cách 2: Ta có số học sinh không giải được A là y, không giải được B là z, không giải
được C là x.
Nếu x  20; y  20; z  20 thì x  y  z  60 . Mâu thuẫn (1).
Do đó trong ba số x, y, z phải có một số không vượt quá 20.
Như vậy có một bài toán mà có nhiều nhất 20 thí sinh không giải được. Do đó bài
toán này có ít nhất 40 thí sinh giải được. Vậy có một bài toán mà có ít nhất 40 thí sinh
giải được.
Ví dụ 15. Giả sử A là một tập con của tập các số tự nhiên N. Tập A có phần tử nhỏ
nhất là 1, phần tử lớn nhất là 100 và mỗi X thuộc A  x  1 , luôn tồn tại a, b cũng

thuộc A sao cho x  a  b (a có thể bằng b). Hãy tìm một tập A có số phần tử nhỏ nhất.
Lời giải
Giả sử A có n số, chúng ta xếp chúng theo thứ tự 1  x1  x 2  x 2    x n  100. 1
Suy ra với mỗi k  1,2,3, ,n  1 ta có xk 1  xi  x j  xk  xk  2xk  2  với 1  i, j  k.
Áp dụng kết quả (2) ta thu được
x 2  1  1  2; x 3  2  2  4; x 4  8; x 5  16; x 6  32; x7  64.
Suy ra tập A phải có ít nhất 8 phần tử.
+ Giả sử n  8  x8  100 .
Vì x6  x7  32  64  96  x8  2x7  x7  50.
Vì x5  x6  16  32  48  x7  2x6  x6  25.
25
Vì x4  x5  8  16  24  25  x6  2x5  x5  (mâu thuẫn).
2
+ Nếu n  9 , ta có tập 1; 2; 3; 5; 10; 20; 25; 50; 100 thỏa mãn yêu cầu bài toán.

Ví dụ 16. Chia 18 vật có khối lượng 2016 2 ; 20152 ; 2014 2 ; ...; 19992 (gam) thành ba
nhóm có khối lượng bằng nhau (không được chia nhỏ các vật đó).
Lời giải
Ta xó nhận xét:
2
n 2   n  5   2n 2  10n  25  X  25
2 2
 n  1   n  4   2n 2  10n  17  X  17
2 2
 n  2   n  3  2n 2  10n  13  X  13

12
Lần thứ nhất chia 6 vật có khối lượng 1999 2 ,20002 , 20012 ,20022 , 20032 , 20042 thành ba
phần như sau:
2
1999 2  20042  1999 2   1999  5   2.1999 2  10.1999  25  A  25
2 2
2000 2  20032   1999  1   1999  4   2.1999 2  10.1999  9  A  17
2 2
20012  2002 2   1999  2    1999  3   2.1999 2  10.1999  13  A  13
Lần thứ hai chia 6 vật có khối lượng 20052 , 2006 2 , 2007 2 , 2008 2 , 20092 , 2010 2 thành ba phần
gồm:
2
20052  2010 2  20052   2005  5   2.20052  10.2005  25  B  25
2 2
20062  2009 2   2005  1   2005  4   2.20052  10.2005  17  B  17
2 2
2007 2  2008 2   2005  2    2005  3   2.20052  10.2005  13  B  13

Lần thứ ba, chia 6 vật có khối lượng 20112 ,20122 , 20132 , 2014, 2015,2016 2 thành ba phần
như sau:
2
20112  2016 2  20112   2011  5   2.20112  10.2011  25  C  25
2 2
2012 2  20152   2011  1   2011  4   2.20112  10.2011  17  C  17
2 2
20132  20142   2011  2    2011  3   2.20112  10.2011  13  C  13
Nhận thấy:
A  25  B  17  C  13  A  17  B  13  C  25  A  13  B  25  C  17  A  B  C  55
Do đó ta chia theo ba nhóm gồm:
Nhóm 1: A  25  B  17  C  13  19992  20042  20062  20092  20132  20142
Nhóm 2: A  17  B  13  C  25  20002  20032  2007 2  20082  20112  20162
Nhóm 3: A  13  B  25  C  17  20012  20022  20052  20102  20122  20152
Khi đó khối lượng của mỗi nhóm đều bằng A  B  C  55 gam.

Ví dụ 17. Một bảng hình vuông kích thước 10x10. Hỏi có thể điền được các số 1, 2,
3, .. . 99, 100 vào các ô của bảng (mỗi ô điền một số) sao cho 2 tính chất sau đồng
thời được thoả mãn:
(i) Tổng các số trên mỗi hàng, mỗi cột bằng nhau và bằng S
(ii) Với mỗi số k  1; 2; 3;...;10 thì tổng các số ở các ô (i; j) ( ô ở hàng i, cột j)
với i  j  k chia hết cho 10, có tổng bằng S.
Lời giải
1
Giả sử điền được các số thỏa mãn yêu cầu bài toán. Khi đó S 
10
 1  2    100   505
là một số lẻ
Chia các ô  i; j của bảng thành 4 loại:
 Loại 1 gồm các ô mà i; j cùng lẻ, gọi S1 là tổng của tất cả các số trên các ô loại 1.

13
 Loại 2 gồm các ô mà i lẻ, j chẵn, gọi S2 là tổng của tất cả các số trên các ô loại 2.
 Loại 3 gồm các ô mà i chẵn, j lẻ, gọi S3 là tổng của tất cả các số trên các ô loại 3.
 Loại 4 gồm các ô mà i; j cùng chẵn, gọi S 4 là tổng của tất cả các số trên các ô loại
4. Khi đó
+ S1  S 2 là tổng các số trên tất cả các hàng lẻ, nên S1  S 2  5S
+ S 2  S 4 là tổng các số trên tất cả các cột chẵn, nên S 2  S 4  5S
+ Loại 1 và loại 4 đều gồm các ô mà i  j chẵn, do đó S1  S4  5S
Suy ra 2  S1  S 2  S 4   15S . Do S lẻ nên vế phải là số lẻ và vế trái là số chẵn. Điều này
vô lý.
Vậy không thể điền được các số thỏa mãn.

Ví dụ 18. Trong bảng 11x11 ô vuông ta đặt các số tự nhiên từ 1 đến 121 vào các ô
một cách tùy ý (mỗi ô đặt duy nhất một số và hai ô khác nhau đặt hai số khác nhau).
Chứng tỏ rằng tồn tại hai ô vuông kề nhau (tức là hai ô có chung một cạnh) sao cho
hiệu hai số đặt trong hai ô vuông đó lớn hơn 5.
Lời giải
Giả sử không có hai ô vuông kề nhau có hiệu hai số đặt 1 2 3 4 …
trong hai ô đó lớn hơn 5. Đánh dấu số hàng và số cột như 2
hình vẽ xét ô ghi số 1 ở ô  a, b  ứng với hàng a và cột b. 3

Xét ô ghi số 121 ở ô  c,d  4



không mất tính tổng quát ta giả sử c  a; d  b . Đặt xa , b

và xc ,d là các số ghi ở ô  a, b  và  c,d  ta có

121  x c ,d   x c 1,d   5  x c  2,d   10  ...  xa,d   5  c  a   xa ,b   5  c  a   5  d  b 


 1  5  11  1  5  11  1  101

Điều này là vô lí.


Do đó điều giả sử là sai nên ta có điều phải chứng minh.
Ví dụ 19. Trong mặt phẳng tọa độ Oxy, cho tam giác ABC có các đỉnh là các điểm
nguyên (một điểm được gọi là điểm nguyên nếu hoành độ và tung độ của điểm đó là
các số nguyên). Chứng minh rằng hai lần diện tích của tam giác ABC là một số
nguyên.
Lời giải

14
Đặt A  x 2 ; y 2  , B  x 3 ; y 3  , C  x1 ; y1  khi đó P có hoành độ là x1 và D có hoành độ x2,

N có hoành độ là x3 và R có tung độ y2, S có tung độ là y1 và T có tung độ là y3


Ta có
1 1 1
S ABC  S CBNP  S ABND  S ADPC 
2
 y 3  y 2  x 3  x1    y 3  y 2  x 3  x 2    y 2  y1  x 2  x1 
2 2
1
  y x  y 3 x1  y2 x1  y2 x3  y 3 x3  y3 x2  y 2 x3  y 2 x2  y 2 x2  y 2 x1  y1x2  y1x1 
2 3 3
1 1 1
   y 3 x1  y 2 x1  y 3 x 2  y 2 x1 )  x1  y 2  y1   y 3  x 2  x1 
2 2 2

Nên ta được 2S ABC  x1  y 2  y1   y 3  x 2  x1 

Vì các tọa độ là các số nguyên vậy diên tích hai lần diện tích tam giác ABC là số
nguyên

Ví dụ 20. Cho bảng ô vuông 2015  2015 . Kí hiệu ô 1 3 6 10


 i, j là ô ở hàng thứ i, cột thứ j. Ta viết các số 2 5 9 …
nguyên dương từ 1 đến 2015 vào các ô của bảng theo 4 8 …
quy tắc sau: 7 …
i) Số 1 được viết vào ô  1,1 . …
Hình 1
ii) Nếu số k được viết vào ô  i, j ,  i  1 thì số
k  1 được viết vào ô  i  1, j  1 .
iii) Nếu số k được viết vào ô  1, j thì số k  1 được
viết vào ô  j  1,1 . (Xem hình 1)
Khi đó số 2015 được viết vào ô  m, n  . Hãy xác định
m và n.
Lời giải
Theo đề bài, các số nguyên dương được sắp xếp theo từng hàng chéo của bảng: Hàng
chéo thứ nhất có 1 số, hàng chéo thứ hai có 2 số, …
Giả sử số x nằm ở hàng chéo thứ k thì ta có
k  k  1 k  k  1 1  1  8x 1  1  8x  1  1  8x 
x  k k 
2 2 2 2  2 

 1  1  8.2015 
Áp dụng x  2015 ta có k     63 .
 2 
k  k  1
Số đầu tiên ở hàng chéo thứ k  63 là  1  1954
2
15
Như vậy số 2015 nằm ở vị trí thứ 2015  1954  1  62 của hàng chéo thứ 63 (Vị trí áp
chót)
Tọa độ của nó là  2; 62  .
Ví dụ 21. Gọi T là tập hợp các tam giác mà các đỉnh của tam giác có tọa độ nguyên
và các cạnh của tam giác cũng là số nguyên. Chứng minh rằng các tam giác cân thuộc
tập hợp T được chia thành hai tam giác vuông bằng nhau cũng thuộc tập hợp T.
Lời giải
Trong tam giác cân thì đường cao hạ từ đỉnh tam giác cân chia tam giác cân thành hai
tam giác vuông bằng nhau. Xét tam giác ABC có CA  CB  x và AB  y với x, y  N* .
Không mất tính tổng quát ta có thể giả sử tọa độ điểm C là C  0; 0  và

A  a 1 ; a 2  , B  b1 ; b 2  với các số nguyên a1 ; b1 ; a 2 ; b2 .


2 2
Khi đó ta được x2  a12  a 22  b12  b22 và y 2   a1  b1    a 2  b2   2  x 2  a1b1  a 2 b2 

Gọi M là trung điểm của AB, khi đó ta được CM vuông góc với AB.
 a 1  b1 a 2  b 2 
Từ đó tọa độ của điểm M là M  ; .
 2 2 

Từ y 2  2  x 2  a1 b1  a 2 b2  suy ra y2 chia hết cho 4 nên  a1  b1  ;  a 2  b2  đều là số

chẵn.
Do đó a1  b1   a1  b1   2b1 ;a 2  b2   a 2  b2   2b2 đều là các số chẵn.

Từ đó dẫn đến M có tọa độ nguyên.


Từ đó ta được các tam giác vuông CAM và CBM có các đỉnh có toạn độ nguyên.
y
Lại thấy AM  BM  là số nguyên do y là số chẵn.
2
Như vậy ta cần chứng minh được CM là số nguyên.
1 2 2
Ta có CM 2   a1  b1    a 2  b2   nên CM2 là số nguyên do a1  b1 ; a 2  b2 là các số
4 

chẵn.
Ta xét hai trường hợp sau
1 2 a  b2
 Nếu a1  b1  0 , khi đó CM 2 
4
 a 2  b2   CM  2
2
là số nguyên.

16
 a  b1 
2
  a  b 2 
2
 Nếu a 1  b1  0 , khi đó CM   1  1   2 2
 .
 2    a 1  b1  
 

Từ x 2  a12  a 22  b12  b 22 ta được a12  b12    a 22  b22  hay ta được

a 1
 b1  a 1  b1     a 2  b 2  a 2  b 2 

Nếu a 2  b 2  0 thì a1  b1  0 , suy ra A và B trùng nhau, điều này vô lí.


a 2  b2 a  b1
Do đó a 2  b 2  0 , do đó ta được  1 .
a 1  b1 a 2  b2

a b 
2
  a  b 2   a  b 2 y2
2
Từ đó dẫn đến CM   1 1  1   1 1
   
1 1
 .
 2    a 2  b 2    2   a  b 2
  2 2

Suy ra CM2 là bình phương của một số hữu tỉ, mà CM2 là số nguyên nên suy ra CM 2
là số chính phương. Do đó CM là số nguyên. Vậy bài toán được chứng minh xong.
Ví dụ 22. Trong mặt phẳng cho 9 điểm có tọa độ nguyên, trong đó không có 3 điểm
nào thẳng hàng. Hỏi trong số các tam giác được tạo thành từ 3 trong 9 điểm đó có ít
nhất bao nhiêu tam giác có diện tích nguyên?
Lời giải
Với tam giác ABC có tọa độ đỉnh A  x A ; y A  , B  x B ; y B  , C  xC ; y C  thì

1
S ABC   x  xA  y B  y A    x B  xA  yC  y A  (1)
2 C
 Xét 9 điểm A, B, C, D, F, G, H, I có tọa độ nguyên thì tọa độ của mỗi điểm sẽ
thuộc một trong các dạng sau: (chẵn, chẵn), (lẻ, lẻ), (lẻ, chẵn), (chẵn, lẻ). Do đó theo
9
nguyên lí Dirichlets tồn tại ít nhất    1  3 điểm thuộc cùng một dạng, tức là tọa độ
4
cùng tính chẵn lẻ, giả sử đó là A, B, C.
 Với hai điểm A, B có tọa độ cùng tính chẵn lẻ thì y B  y A và x B  x A đều là số chẵn

nên diện tích tam giác có cạnh AB đều nguyên (do(1)). Tương tự diện tích các tam
giác có cạnh là AC, BC đều nguyên.
 Với mỗi 2 trong 3 điểm A, B, C kết hợp với 6 điểm còn lại thì được 6 tam giác có

diện tích nguyên. Vậy có ít nhất 3.6  1  19 tam giác có diện tích nguyên.

17
Ví dụ 23. Có 2008 con gà nhốt vào 1004 cái chuồng, mỗi chuồng có 2 con.Sau mỗi
ngày người ta lại thay đổi vị trí của gà sao cho không có hai con gà nào đã nhốt
chung chuồng trước đólại nằm cùng chuồng lần nữa. Hỏi tối đa có bao nhiêu ngày
làm được như vậy?
Lời giải

Vẽ đa giác đều 2007 cạnh nội tiếp trong đường tròn. Ký hiệu tâm là 2008 và các đỉnh
là 1, 2, 3, ….2007. Ký hiệu đoạn thẳng nối i với j là i – j với i, j  1, 2008 .
Xét bán kính 1  2008 . Do tính chất đa giác đều nên ta thấy ngay 1003 dây cung
vuông góc với bán kính ấy là 2 – 2007; 3 – 2006; 4 – 2005;...;1004 – 1005
Xét bán kính 2  2008 . Tương tự ta có 1003 dây cung vuông góc với bán kính ấy là
1  3; 2007 – 4; 2006 – 5;...

Xét bán kính 3  2008 . Tương tự ta có 1003 dây cung vuông góc với bán kính ấy là
2 – 4;1 – 5; 2007 – 6;....

Cứ làm như vậy và cuối cùng xét bán kính 2007  2008 , ta có 1003 dây cung vuông
góc với bán kính ấy là 2006 – 1; 2005 – 2; 2004 – 3;....
Dựa vào nhận xét trên và cách đánh số các con gà từ 1  2008 , ta chỉ ra cách xếp gà
theo yêu cầu bài toán như sau:
+ Ngày thứ nhất xếp vào chuồng các đôi gà như sau: 1 – 2008; 2  2007; ...;1004 – 1005
+ Ngày thứ hai xếp vào chuồng các đôi gà như sau:
2  2007;1  3; 2007 – 4;...;1006 – 1005

+ Ngày thứ ba xếp vào chuồng các đôi gà như sau: 3 – 2008; 2  4;1  5;....;1007 – 1006
………………………
+ Ngày thứ 2007 xếp vào chuồng các đôi gà như sau:
2007 – 2008; 2006  1;...;1004 – 1003

18
Mặt khác không có qua 2007 ngày vì mỗi con gà chỉ có thể cùng chuồng với một trong
2007 con gà còn lại. Vậy tối đa có 2007 ngày để xếp gà theo yêu cầu của đề bài.
Ví dụ 24. Có 9 chiếc bàn vừa màu xanh vừa màu đỏ xếp thành một hàng dọc cách
đều nhau. Chứng minh rằng có ít nhất một chiếc bàn được xếp cách 2 bàn cùng màu
với mình một khoảng cách như nhau.
Lời giải
+ Gọi tên theo thứ tự 9 chiếc bàn là B1 ; B2 ; B3 ; B4 ; B5 ; B6 ; B7 ; B8 ; B9 . Giả sử không có bàn
nào được xếp cách đều hai bàn cùng màu với mình.
+ Không mất tổng quát ta giả sử B5 là bàn màu xanh, khi đó B4 và B6 không thể cùng
màu xanh. Có hai khả năng:
 B4 và B6 cùng màu đỏ. Do đó B4 cách đều B2 và B6, còn B6 cách đều B4 và B8 nên

B2 và B8 cùng màu xanh, suy ra B5 được xếp cách đều hai bàn cùng màu xanh là B2
và B8, trái với giả thiết.
 B4 và B6 khác màu, không mất tổng quát, giả sử B4 màu xanh còn B6 màu đỏ. Do

B4 cách đều B3 và B5 nên B3 là bàn màu đỏ. Do B6 cách đều B3 và B9 nên B9 là bàn
màu xanh. Do B5 cách đều B1 và B9 nên B1 màu đỏ. Do B2 cách đều B1 và B3 nên B2
màu xanh. Do B5 cách đều B2 và B8 nên B8 có màu đỏ. Do B6 và B8 cùng có màu đỏ
nên B7 có màu xanh.
Như vậy B7 được xếp cách đều hai bàn cùng màu xanh là B5 và B9, trái với giả thiết
Vậy cả hai khả năng trên đều dẫn đến vô lý nên điều giả sử là sai.
Như vậy có ít nhất một bàn được xếp cách đều với hai bàn cùng màu với mình.

Ví dụ 25. Mỗi ô vuông đơn vị của bảng hữu hạn m  n (m, n là các số nguyên
dương) được ghi một số thực bất kì. Xét quy tắc biến đổi sau: Mỗi lần đổi dấu tất cả
các số trên một hàng hoặc một cột. Chứng minh rằng sau một số hữu hạn bước thực
hiện quy tắc trên, ta thu được một bảng mà tổng các số trong mọi hàng và mọi cột
đều không âm.
Lời giải
Xét tổng các số trong bảng khi thực hiện phép đổi dấu. Tổng này sẽ tăng khi
tổng các số trên hàng (hoặc cột) trước khi đổi dấu là âm; tổng không đổi khi khi tổng
19
các số trên hàng (hoặc cột) trước khi đổi dấu bằng 0 và tổng này sẽ giảm khi tổng các
số trên hàng (hoặc cột) trước khi đổi dấu là dương.
Khi thực hiện phép biến đổi trên thì mỗi số trong một ô hoặc là trùng với số
ban đầu hoặc là số đối của số ban đầu, vì thế khi thực hiện phép biến đổi trên thì có
thể nhận được tối đa 2 m.n bảng khác nhau, do vậy tổng tất cả các số trong bảng chỉ
nhận hữu hạn giá trị khác nhau.
Ta xét bảng ban đầu: Nếu không có hàng nào và không có cột nào có tổng các
số là âm thì bài toán được chứng minh.
Ngược lại, ta chọn một hàng hoặc cột có tổng các số là âm, ta thực hiện phép
biến đổi trên hàng hoặc cột đó. Khi đó trong bảng vừa nhận được nếu có hàng hoặc
cột có tổng các số là âm thì ta lại thực hiện phép biến đổi trên hàng hoặc cột đó và
nhận được bảng mới và cứ tiếp tục như vậy.
Như vậy mỗi lần thực hiện phép biến đổi thì tổng các số trong bảng tăng lên.
Do tổng đó chỉ nhận hữu hạn giá trị nên hoặc là tại một bước thực hiện nào đó
ta nhận được bảng cần tìm, hoặc là nhận được bảng có tổng đạt giá trị lớn nhất.
Bảng có tổng đạt lớn nhất thỏa mãn bài toán vì nếu trong nó có một vệt có tổng
là âm thì thực hiện phép biến đổi trên vệt đó ta được bảng mới có tổng tăng lên lớn
hơn tổng lớn nhất đó, vô lý.
Vậy trong mọi trường hợp ta đều có điều phải chứng minh.

Ví dụ 26. Một thùng đựng n lít rượu (n là số nguyên dương). Người ta muốn đong hết
để làm rỗng thùng rượu mà chỉ dùng hai bình: một bình có dung tích là 1 lít và bình
kia có dung tích là 2 lít; mỗi thao tác đong chỉ dùng một loại bình. Gọi S(n) là số cách
đong theo thứ tự các thao tác đong để làm rỗng thùng đựng n lít rượu. Hãy liệt kê các
cách đong đó để tính S(1), S(2), S(3), S(4), S(5), S(6). Từ đó rút ra quy luật để tính
S(n) (không cần chứng minh). Áp dụng để tính S(10).

Lời giải
Ta có bảng sau:

20
n Các cách đong để làm rỗng thùng Số cách đong S(n)
rượu
1 {1} (chỉ dùng bình 1 lít) S(1) = 1
2 {1+1; 2} S(2) = 2
3 {1+1+1; 2+1; 1+2} S(3) = 3
4 {1+1+1+1; 2+1+1; 1+2+1; 1+1+2; S(4) = 5
2+2}
{1+1+1+1+1; 2+1+1+1; 1+2+1+1;
5 1+1+2+1; S(5) = 8
1+1+1+2; 2+2+1 2+1+2; 1+2+2}
{1+1+1+1+1+1; 2+1+1+1+1;
1+2+1+1+1;
1+1+2+1+1; 1+1+1+2+1;
6 1+1+1+1+2; S(6) = 13
2+2+1+1; 1+2+2+1; 1+2+1+2;
2+1+2+1;
2+1+1+2; 1+1+2+2; 2+2+2}

Từ kết quả trên ta suy ra: với n  3 thì S(n)  S(n  2)  S(n  1) .

Dãy số S  1 , S  2  , S  3  , ..., S  n  chính là dãy số Fibonacci.

Suy ra S  7   21; S  8   34; S  9   55; S  10   89 .

Ví dụ 27. Cho tập hợp A  0;1; 2; 3; 4; 5; 6; 7; 8; 9 . Chứng minh rằng với mỗi tập con B

gồm năm phần tử của tập hợp A thì trong các tổng x  y với x, y  B và x  y luôn tồn
tại ít nhất hai tổng có chữ số hàng đơn vị giống nhau.
Lời giải
Ta có A  C  D, trong đó C  0; 2; 4; 6; 8 , D  1; 3; 5; 7; 9 và C  D  . Như

vậy với mỗi tập con B gồm 5 phần tử của tập A, luôn tồn tại ít nhất 3 phần tử có cùng
tính chẵn lẻ.

21
Trường hợp 1: Tập B có 3 phần tử cùng tính chẵn (lẻ) và 2 phần tử còn lại cùng tính
lẻ (chẵn).
Nếu 3 phần tử của B cùng tính chẵn là a, b, c và 2 phần tử cùng tính lẻ là d, e
thì 6 tổng: d  a,d  b,d  c,e  a,e  b,e  c cùng tính lẻ, nên phải có hai tổng có chữ số
hàng đơn vị như nhau.
Nếu B có 3 phần tử cùng tính lẻ và 2 phần tử còn lại cùng tính chẵn thì chứng
minh tương tự.
Trường hợp 2: Tập hợp B có ít nhất 4 phần tử cùng tính chẵn (hoặc cùng tính lẻ), gọi
4 phần tử đó là a, b, c, d thì 6 tổng: a  b,a  c,a  d, b  c, b  d,c  d luôn cùng tính chẵn,
nên có 2 tổng có chữ số hàng đơn vị như nhau.
Vậy yêu cầu bài toán được chứng minh.
Ví dụ 28. Cho đa giác lồi A1A 2  A100 . Tại mỗi đỉnh Ak ( k  1, 2,...,100 ), người ta ghi
một số thực a k sao cho giá trị tuyệt đối của hiệu hai số trên hai đỉnh kề nhau chỉ bằng
2 hoặc 3. Tìm giá trị lớn nhất có thể được của giá trị tuyệt đối của hiệu giữa hai số ghi
trên mỗi cặp đỉnh của đa giác đã cho, biết rằng các số ghi tại các đỉnh đã cho đôi một
khác nhau.

Lời giải
Xét đa giác lồi A1A 2  A100 như hình vẽ. Khi đó A1
A2

a k  a k 1  2 hoặc a k  a k 1  3 ( k  1, 2,...,99 ).
A100
A3
Không mất tính tổng quát, coi a1 là nhỏ nhất và a n
A4
là lớn nhất (dễ thấy n  2 ). Đặt d  max a i  a j khi
i j

đó d  a n  a1 . Ta sẽ chứng minh d  149.


Nằm giữa A1 , A n , theo chiều kim đồng hồ có n  2 đỉnh và có 100  n đỉnh, theo chiều
ngược kim đồng hồ. Hơn nữa giá trị tuyệt đối của hiệu giữa hai số kề nhau không
vượt quá 3.
Do đó d  a1  a n  a1  a 2  a 2  a 3  ...  a n 1  a n  3  n  1

Tương tự ta có d  3 100  n  1 . Suy ra d 


 3(n  1)    3(100  n  1)   300  150
2 2
+ Ta có d  150 khi và chỉ khi hiệu giữa hai số ghi trên hai đỉnh kề nhau đúng bằng 3
22
Từ đó ta có a i  a i1  3, i  1, 2,...,99
a  a i  1  a i  1  a i  2
 a i  a i 1  a i 1  a i  2   i  i  1,...,98 
a i  a i  2
 a 1  a100  a 1  a 2  a 2  a 3  ...  a 99  a100  99  a 1  a 2   a 1  a 100  99  a 1  a 2   3  99.3 Đ
iều này không xảy ra suy ra d  150 không thỏa mãn.
Ta xây dựng một trường hợp cho d  149 như sau: a1  0,a 2  2,a k  a k1  3
với k  2, 3, , 52; a 53  a 52  2,a k  a k 1  3, k  54, 55, ,100
Khi đó hiệu lớn nhất a 53  a1  149 . Các số a 2 ,a 3 , ,a 53 có dạng 2  3t , các số
a 54 ,a 55 , ,a100 có dạng 147  3t . Rõ ràng không tồn tại k, t sao cho
2  3t  147  3k  3  k  t   145 ( k, t   ).
Suy ra điều phải chứng minh.
Ví dụ 29. Giả sử A là một tập con của tập các số tự nhiên . Tập A có phần tử nhỏ
nhất là 1, phần tử lớn nhất là 100 và mỗi X thuộc A  x  1 , luôn tồn tại a, b cũng
thuộc A sao cho x  a  b (a có thể bằng b). Hãy tìm một tập A có số phần tử nhỏ
nhất.
Lời giải
Giả sử A có n số, chúng ta xếp chúng theo thứ tự 1  x1  x 2  x 2    x n  100. 1
Suy ra với mỗi k  1,2,3, ,n  1 ta có xk1  xi  x j  xk  xk  2xk  2  với 1  i, j  k.
Áp dụng kết quả (2) ta thu được
x 2  1  1  2; x 3  2  2  4; x 4  8; x 5  16; x 6  32; x7  64.
Suy ra tập A phải có ít nhất 8 phần tử.
+ Giả sử n  8  x8  100 .
Vì x6  x7  32  64  96  x8  2x7  x7  50.
Vì x5  x6  16  32  48  x7  2x6  x6  25.
25
Vì x 4  x 5  8  16  24  25  x6  2x5  x 5  (mâu thuẫn).
2
+ Nếu n  9 , ta có tập 1; 2; 3; 5; 10; 20; 25; 50; 100 thỏa mãn yêu cầu bài toán.
Ví dụ 30. a) Trên một tấm ván hình chữ nhật có ba cái cọc. Ở một cọc có n  3 cái
đĩa hình tròn có đường kính nhỏ dần từ dưới lên (hình bên). Số lần ít nhất cần có để
chuyển cả 3 đĩa sang một cọc khác là bao nhiêu ? Biết rằng mỗi lần chỉ di chuyển một
đĩa sang cọc khác sao cho đĩa với đường kính lớn luôn nằm dưới đĩa có đường kính
bé hơn.
b) Theo quy tắc như trên, số lần ít nhất cần có để chuyển chồng đĩa có
n  4 đĩa; có n  10 đĩa sang một cọc khác là bao nhiêu ? Hãy viết công thức tổng
quát để tính số lần ít nhất cần có để chuyển chồng n đĩa sang một cọc khác với n là
số tự nhiên bất kì lớn hơn 2.
Lời giải
a) Để chuyển chồng n  3 đĩa từ cọc số 2 sang một cọc
khác:
1 2 3
23
- Bước 1: Trước hết di chuyển 2 đĩa trên cùng sang một cọc khác (ví dụ cọc số 1) với
số thao tác: Di chuyển đĩa trên cùng (đĩa A) sang cọc 3; di chuyển đĩa thứ hai (đĩa B)
sang cọc 1; cuối cùng chuyển đĩa A sang cọc 1. Do đó số lần di chuyển tối thiểu là 3.
- Bước 2: Chuyển đĩa lớn nhất (đĩa C) sang cọc 3 gồm một thao tác.
- Bước 3: Chuyển chồng 2 đĩa từ cọc 1 sang cọc 3: Chuyển đĩa A sang cọc 2; chuyển
đĩa B sang cọc 3 và chuyển đĩa A sang cọc 3. Số lần di chuyển tối thiểu là 3.
Vậy số lần ít nhất cần có để chuyển cả 3 đĩa sang một cọc khác 3  1  3  7  2 3  1 .
b) Trường hợp chồng đĩa có n  4 đĩa:
- Bước 1: Chuyển chồng 3 đĩa trên cùng sang cọc khác
(ví dụ cọc 1). Số lần di chuyển tối thiểu từng đĩa là
1 2 3
23  1 .

- Bước 2: Chuyển đĩa lớn nhất sang cọc khác (cọc số 3) gồm 1 thao tác.
- Bước 3: Chuyển chồng 3 đĩa từ cọc 1 sang cọc 3. Số lần di chuyển tối thiểu từng đĩa
là 2 3  1 .
Vậy: Số lần ít nhất cần có để chuyển chồng 4 đĩa sang một cọc khác là:

2 3
  
 1  1  2 3  1  2.2 3  1  2 4  1  15

Tương tự thì số lần ít nhất cần có để chuyển chồng 5 đĩa sang một cọc khác là:

2 4
  
 1  1  2 4  1  2.2 4  1  2 5  1  31

. . .
Số lần ít nhất cần có để chuyển chồng 10 đĩa sang một cọc khác là:

2 9
  
 1  1  2 9  1  2.2 9  1  210  1  1023

Tổng quát: Số lần ít nhất cần có để chuyển chồng n đĩa  n  , n  2  sang một cọc

khác là:

2 n 1
  
 1  1  2 n 1  1  2.2 n 1  1  2 n  1

Ví dụ 31. Với mỗi số nguyên n lớn hơn hoặc bằng 2 cố định xét các tập n số thực đôi
một khác nhau X  x1 , x 2 ,..., x n  . Kí hiệu C  X  là số giá trị khác nhau của tổng

x i  x j ,  1  i  j  n  . Tìm giá trị nhỏ nhất và lớn nhất của C  X 

24
Lời giải
Giả sử các phần tử của tập hợp X được sắp thứ tự là x1  x2  x3  ...  x n .
Ta có x1  x2  x1  x3  ...  x1  xn  x 2  x n . Đối với tập hợp n số thực bất kì ta

luôn có ít nhất  n  1   n  2   2n  3 giá trị phân biệt các tổng xi  x j . Do đó

C  X   2n  3 .

Xét tập hợp X 1  1; 2; 3; ...; n , với 1  i  j  n thì x i  x j  i  j  3; 4;...; 2n  1

suy ra C  X   2n  3 . Vậy ta có Min C X   2n  3 .

n  n  1 n  n  1
Số các tổng x i  x j ,  1  i  j  n  bằng , suy ra C  X  
2 2

Xét tập hợp X 2  2; 22 ; 2 3 ;...; 2 n  thì với mọi 1  i  j  n ta có x i  x j  2 i  2 j .


Giả sử tồn tại 1  r  s  n sao cho xi  x j  xr  xs suy ra
   
2 i  2 j  2 r  2 s  2 r 1  2 s  r  2 i 1  2 ji do đó ta được 2i  2r ; 2r  2i  i  r  j  s nên ta

n  n  1
được C  X 2  
2
n  n  1
Vậy MaxC X   .
2
Ví dụ 32. Trong mỗi ô bàn cờ kích thước 5x5 có một con bọ dừa. Vào một thời điểm
nào đó tất cả các con bọ dừa bò sang ô bên cạnh (ngang hoặc dọc). Có thể khẳng định
rằng sau khi các con bọ dừa di chuyển sẽ luôn có ít nhất một ô trong bàn cờ không có
con bọ dừa nào trong đó không?
Lời giải
Ta tô đen - trắng các ô bàn cờ như hình vẽ. Khi đó số ô đen
nhiều hơn số ô trắng. Như vậy số con bọ dừa ở ô đen sẽ
nhiều hơn số con bọ dừa ở ô trắng. Do mỗi con bọ dừa chỉ di
chuyển sang ô bên cạnh(ngang hoặc dọc), vì thế sau khi di
chuyển các ô đen sẽ chứa các con bọ dừa ở ô trắng. Mà số
con bọ dừa ở ô đen nhiều hơn số con bọ dừa ở ô trắng nên
sau khi các con bọ dừa bò đi sẽ có ít nhất một ô đen bị bỏ
trống .

25
Vậy có thể khẳng định rằng sau khi di chuyển sẽ luôn có ít nhất một ô trong bàn cờ
không có con bọ dừa nào trong đó.
Ví dụ 33. Trên lưới ô vuông cạnh 1. Người ta tô màu các ô bằng 2 màu đen trắng xen
kẽ. Tính bán kính lớn nhất của đường tròn chỉ đi qua ba ô trắng.
Lời giải
 Xét đường tròn chỉ thuộc một ô trắng: Đường
kính của nó bằng 1.
 Xét đường tròn đi qua nhiều ô trắng nên
đường tròn đó phải đi qua các đỉnh của ô trắng.
Không mất tính tổng quát, giả sử đường tròn đi
qua đỉnh A của ô trắng ABCD.
+ Nếu đường tròn đi qua 2 đỉnh liên tiếp
của ô trắng ( A, B). Khi đó ta lại xét hai trường
hợp:
 2
 Đường tròn qua A, B, E. Khi đó nó là đường tròn  I;
 2 
 
 10 
 Đường tròn qua A, B, G. Khi đó nó là đường tròn  K; 
 2 

+ Nếu đường tròn qua 2 đỉnh đối diện của ô trắng, không mất tính tổng quát ta giả sử
là hai đỉnh đó là A và C. Ta lại xét hai trường hợp:
 Đường tròn ua A, C, M(Tương tự qua A, B, G)
 Đường tròn qua A, C, N (Tương tự qua A, B, G)
10
Cả hai trường hợp trên bán kính của đường tròn là .
2
Từ các kết quả trên ta suy ra bán kính lớn nhất của đường tròn thỏa mãn yêu cầu đề
10
bài là .
2
Ví dụ 34. Một hình vuông cạnh 4cm bị khuyết 2 hình vuông đơn vị (cạnh 1cm) ở hai
góc đối diện nhau. Một người tìm cách dùng 7 quân đôminô hình chữ nhật kích thước
1cm x 2cm để đặt vào phần còn lại của hình vuông bị khuyết này. Hỏi có thể làm
được không?
Lời giải
Phần còn lại của hình vuông gồm 4x4  2  14 hình vuông đơn vị. Ta tô màu 14 hình
vuông này bằng hai màu đen trắng kế tiếp như bàn cờ vua. Ta thấy số ô màu đen là 6
còn số ô màu trắng là 8. Mỗi quân đôminô khi đặt vào bàn cờ sẽ lấp vào 1 ô đen và 1
ô trắng liền kề. Vì số ô đen khác số ô trắng nên không thể thực hiện được việc đặt 7
26
quân đôminô vào hình vuông bị khuyết này.
Nhận xét. Nếu không dùng lập luận trên, ta có thể thử lát theo các cách khác nhau và
đều không thể thực hiện được. Bất biến ở đây là số ô đen và trắng của mỗi cây
đôminô là bằng nhau.
Ví dụ 35. Vùng nọ có khu đất vàng 100 × 100 m, chia ra làm 100 lô, mỗi lô 10 × 10
m. Vua bãi rác muốn lấn chiếm khu đất này nên sai tay chân đổ rác vào một số ô.
Nếu một ô nào chưa có rác mà có ít nhất hai ô cạnh nó (có chung cạnh) đã bị đổ rác
thì (đáng tiếc) hôm sau nhân dân cũng sẽ đổ rác vào ô đó. Nếu đến một ngày nào đó
tất cả các ô đều bị đổ rác thì vua bãi rác sẽ chiểm khu đất. Nếu vua bãi rác muốn
chiếm khu đất này thì lúc đầu cần đổ rác vào ít nhất mấy ô?
Lời giải

Ta đưa bài toán về mô hình bảng ô vuông kích thước 10×10 và mỗi ô bị đổ rác thì sẽ
được tô đen. Mỗi bước biến đổi tương ứng với việc ta tô đen một ô chưa được tô và
chung cạnh với ít nhất hai ô được tô đen. Do đó, ta chỉ cần tìm số ô đen được tô ban
đầu sao cho sau một số hữu hạn bước biến đổi, ta có thể tô đen cả bảng. Gọi p là chu
vi của tất cả các phần được tô đen. Ta chứng minh rằng sau mỗi bước biến đổi thì p
không tăng. Thật vậy, do ở mỗi bước biến đổi ta tô đen một ô khi nó phải chung cạnh
với ít nhất hai ô đen. Ta có các trường hợp sau: Chu vi không đổi, chu vi giảm 2 đơn
vị, chu vi giảm 4 đơn vị.
Khi cả bảng được tô đen thì p'  10  4  40 . Do đó, để tô đen cả bảng ban đầu
p
p  p'  40 . Mà mỗi ô có chu vi là 4. Suy ra ban đầu cần tô ít nhất  10 ô. Ta tô
4
mười ô trên cùng một đường chéo chính thì sau hữu hạn bước biến đổi thì cả bảng sẽ
27
được tô đen. Ban đầu vua cho đổ rác vào mười ô trên một đường chéo chính số 1.
Ngày hôm sau các ô trên hai đường chéo chính số 2 sẽ bị đổ rác (do mỗi ô đều kề hai
ô ở đường chéo số 1). Tiếp tục như vậy thì đến ngày thứ 10 cả khu đất sẽ bị đổ rác.
Vậy ta cần mỗi ngày chiếm ít nhất 10 ô.

Ví dụ 36. Cho hình hộp chữ nhật có độ dài ba kích thước là các số tự nhiên. Các mặt
của hình hộp được sơn màu xanh. Chia hình hộp này thành các khối lập phương đơn
vị bằng các mặt phẳng song song với các mặt của hình hộp. Tìm các kích thước của
hình hộp , biết rằng số các khối lập phương đơn vị không có mặt nào màu xanh bằng
1
tổng số các khối lập phương đơn vị.
3
Lời giải
Giả sử độ dài 3 cạnh hình hộp chữ nhật là x  y  z .
1
Từ điều kiện bài toán ta suy ra x  3 và  x  2  y  2  z  2   xyz
3
3


x2 y2 z2
  nên khi x  7 ta có
 x  2  y  2  z  2    5   1
x y z xyz 7 3
 

Do đó x  6 6, suy ra x  3; 4; 5; 6 , Ta xét các trường hợp sau

+ Nếu x  3 thì  y – 2  z – 2   yz , điều này không thể xảy ra.

4
+ Nếu x  4 , ta có 2  y – 2  z – 2   yz   y – 6  z – 6   24
3

Lúc đó  x;y;z    4;7;30  ,  4;8;18  ,  4;9;14  ,  4;10; 12 

5
+ Nếu x  5 , ta có 3  y – 2  z – 2   yz   2y – 9  2z – 9   45
3

Lúc đó  x; y; z    5; 5; 27  ,  5; 6;12  ,  5; 7; 9 

+ Nếu x  6 , ta có 4  y – 2  z  2   2yz   y  4  z – 4   8 . Ta có nghiệm duy nhất

 6; 6; 8 
Tóm lại có 8 kết quả cho ba kích thước hình họp chữ nhật
 x;y;z    4;7;30  ,  4;8;18  ,  4;9;14  ,  4;10; 12  ,  5; 5; 27  ,  5; 6;12  ,  5; 7; 9  ,  6; 6; 8 
Ví dụ 37. Cho tam giác ABC vuông cân tại A có độ dài cạnh huyền bằng 2015.
Trong tam tam giác ABC lấy 2031121 điểm phân biệt bất kỳ. Chứng minh rằng tồn
tại ít nhất hai điểm có khoảng cách không lớn hơn 1.
28
Lời giải
Chia cạnh huyền BC thành 2015 đoạn thẳng bằng nhau, mỗi đoạn thẳng có dộ dài
bằng 1. Từ các điểm chia đó vẻ các đường thẳng song song với hai cạnh AB và AC ta
được 2015 tam giác vuông cân có cạnh huyền bằng 1 và  2014  2013  1 hình
vuông có đường chéo bằng 1.
1
Do đó trong tam giác ABC có tất cả 2015  2014.2015  2031120 hình (vừa hình
2
vuông có đường chéo bằng 1 vừa tam giác vuông cân có cạnh huyền bằng 1).
Như vây theo nguyên lý Dirchhlets thì trong 2031121 điểm sẽ tồn tại ít nhất hai điểm
nằm trong một hình nào đó.
Với hai điểm đó thì khoảng cách của nó không lớn hơn 1.

Ví dụ 38. Mọi điểm trên mặt phẳng được đánh dấu bởi một trong hai dấu (+) hoặc
(  ). Chứng minh rằng luôn chỉ ra được 3 điểm trên mặt phẳng làm thành tam giác
vuông cân mà ba đỉnh của nó được đánh cùng dấu.
Lời giải
Chứng minh rằng luôn chỉ ra được 3 điểm trên C D

mặt phẳng làm thành tam giác vuông cân mà ba đỉnh


của nó được đánh cùng dấu. Dựng tam giác vuông cân I

ABC đỉnh A. Do chỉ đánh bởi hai dấu (+), (  ) nên tồn
tại hai điểm cùng dấu , không mất tổng quát giả sử hai A B

điểm A, B cùng dấu và cùng dấu (+).


 Nếu C có dấu (+) thì tam giác vuông cân ABC là tam

giác phải tìm.


 Nếu C có dấu (  ) thì ta dựng điểm D sao cho ABDC

là hình vuông.
+ Nếu D có dấu (+) thì tam giác ABD là tam giác cần tìm.
+ Nếu D có dấu (  ) thì gọi I là giao điểm của AD và BC .
- Nếu I có dấu (+) thì tam giác vuông cân ABI là tam giác cần tìm.
- Nếu I dấu (  ) thì dễ thấy tam giác vuông cân CID có ba đỉnh cùng dấu (  ) là
tam giác cần tìm.

29
Ví dụ 39. Cho một đa giác lồi có diện tích k  cm 2  . Chứng minh rằng tồn tại một hình

bình hành có diện tích không vượt quá 2k  cm 2  chứa toàn bộ đa giác.

Lời giải
Gọi C là đỉnh cách xa AB nhất (hình vẽ).
 Trường hợp 1: Nếu AC là đường chéo của đa giác lồi.

Qua C kẻ a // b  A, B  b  . Gọi D, E là các đỉnh cách xa

AC nhất, qua D kẻ đường thẳng d // AC, qua E kẻ đường


thẳng c // AC. Gọi MNPQ là hình bình hành tạo bởi a ,b,c,d
suy ra các đỉnh của đa giác nằm trong hoặc trên biên của
hình bình hành MNPQ.
Ta chứng minh SMNPQ  2k  cm 2  . Thật vậy, gọi Sd là diện tích đa giác

1
Ta có S ACD  S ACE  Sd  S MNPQ  Sd  S MNPQ  2k  cm 2 
2
 Trường hợp 2: Nếu AC là cạnh của đa giác lồi. Gọi E là đỉnh cách xa AC nhất ta

chứng minh tương tự.


Vậy bài toán được chứng minh.
  120 0 , trên tia phân giác Oz của góc xOy lấy điểm A sao cho
Ví dụ 40. Cho góc xOy
độ dài đoạn thẳng OA là một số nguyên lớn hơn 1. Chứng minh rằng luôn tồn tại ít
nhất ba đường thẳng phân biệt đi qua A và cắt hai tia Ox, Oy lần lượt tại B và C sao
cho độ dài các đoạn thẳng OB và OC đều là các số nguyên dương.
Lời giải
O

A
B C

y
x
z

30
Qua điểm A vẽ đường thẳng d1 vuông góc với OA. Khi đó tam giác OAB vuông tại O
  600 , từ đó ta được OB  2AO . Đặt OA  a  1 với a nguyên, khi đó ta
và có AOB
được OB là số nguyên dương lớn hơn 1. Hoàn toàn tương tự ta có OC là số nguyên
dương lớn hơn 1. Từ đó đường thẳng d1 thỏa mãn yêu cầu bài toán.
Trên tia Ox lấy điểm B sao cho OB  a  1 nguyên dương. Đường thẳng d 2 đi qua A,
B cắt tia Oy tại C. Do OA là đường phân giác của tam giác OBC nên ta chứng minh
1 1 1
được   .
OB OC OA
1 1 1
Từ đó suy ra    OC  a  a  1 là số nguyên dương. Do đó đường thẳng
a  1 OC a
d 2 là một đường thẳng cần tìm.

Hoàn toàn tương tự lấy B trên Ox sao cho OB  a  a  1 , khi đó đường thẳng d 3 đi qua

điểm A và B cắt tia Oy tại C, chứng minh đương tự như trên ta được OC  a  1 là số
nguyên dương lớn hơn 1. Như vậy đường thẳng d 3 là một đường thẳng cần tìm.
Dễ thấy ba đường thẳng d1 ,d2 ,d3 phân biệt. Từ đó bài toán được chứng minh.

Ví dụ 41. Chứng minh rằng nếu ba điểm A, B, C không có điểm nào nằm bên ngoài
đường tròn (O) sao cho ABC có ba góc nhọn thì chu vi của đường tròn ngoại tiếp
ABC không lớn hơn chu vi (O)
Lời giải
Gọi đường tròn ngoại tiếp tam giác ABC là (I), I nằm trong tam giác ABC.
- Nếu A, B, C nằm trên đường tròn (O) thì hai đường tròn (I) và (O) trùng nhau.
- Nếu (O) đựng (I) hoặc (O) tiếp xúc trong với (I) thì chu vi của (I) nhỏ hơn chu vi
của (O).
- Nếu (O) và (I) cắt nhau tại M và N. Vì tam giác ABC nhọn nên số đo cung nhỏ MN
nhỏ hơn 1800 và số đo cung lớn MN lớn hơn 1800 , do đó tồn tại đường kính của (I)
nằm trong (O). Do đó chu vi của (I) nhỏ hơn chu vi của (O).
Ví dụ 42. Mỗi điểm của mặt phẳng được tô bởi một trong ba màu đỏ, xanh, vàng.
Chứng minh rằng tồn tại hai điểm A và B được tô bởi cùng một màu mà độ dài
AB  1

31
Lời giải
+ Giả sử trái lại, với mọi cách tô, không tồn tại hai điểm cùng màu mà có khoảng
cách bằng 1. Xét hai điểm M và N cách nhau một khoảng là 3 , khi đó tồn tại các

điểm P và Q sao cho các tam giác MPQ và NPQ là các tam giác đều có độ dài cạnh
bằng 1. Khi đó, do hai điểm có khoảng cách bằng 1 thì được tô bởi hai màu khác
nhau, nên M và N phải được tô bởi cùng một màu, chẳng hạn nếu tô P màu đỏ và Q
màu vàng thì M, N phải tô cùng màu xanh.
+ Từ đó nếu điểm M được tô màu Xanh, thì mọi điểm nằm trên đường tròn tâm M,
bán kính 3 đều được tô màu Xanh. Nhưng trên đường tròn này luôn có hai điểm
mà khoảng cách giữa chúng bằng 1. Mâu thuẫn với giả thiết phản chứng.
Từ đó suy ra điều phải chứng minh.

Ví dụ 43. Trong bảng hình vuông gồm 10  10 ô vuông (10 hàng, 10 cột), người ta
viết vào các ô vuông các số tự nhiên từ 1 đến 100 theo cách như sau: Ở hàng thứ
nhất, từ trái sang phải, viết các số từ 1 đến 10; ở hàng thứ hai, từ trái sang phải, viết
các số từ 11 đến 20; cứ như vậy cho đến hết hàng thứ 10. Sau đó cắt bảng hình vuông
thành những hình chữ nhật nhỏ kích thức 1x2 hoặc 2x1 . Tính tích số của hai số trong
mỗi hình chữ nhật nhỏ rồi cộng 50 tích lại. Cần phải cắt hình vuông như thế nào để
tổng tìm được nhỏ nhất ? Hãy tính giá trị nhỏ nhất đó.
Lời giải
Cắt hình vuông thành các hình chữ nhật cỡ 1x2 hoặc 2x1 thì được tất cả 50 hình. Giả
sử trong hình thứ k có 2 số a k ; bk thì hoặc a k  bk  1 hoặc a k  bk  10
2
a 2  b2  a  b 
Ta có a k .bk  k k  k k , từ đó suy ra
2 2
a 1 .b1  a 2 .b 2  ...  a 50 .b 50
1 2 1 2 2 2
 a 1  b12  a 22  b 22  ...  a 250  b 250    a 1  b1    a 2  b 2    a 50  b 50  
     
2   2 

1 2 1
Trong đó
2     
a 1  b12  a 22  b 22  ...  a 50
2
2 
 b 50
 2
  
 11  2 2    100 2  169175 và mỗi số

2
a k
 b k  hoặc bằng 1 hoặc bằng 100.

2
Do đó để tổng thu được là nhỏ nhất thì  a k  b k   100, k  1, 2,..., 500.

32
Vì vậy cần cắt hình vuông thành các hình chữ nhật với kích thước 2x1 . Và khi đó giá
trị nhỏ nhất của tổng bằng 169175  25  100  166675 .
Ví dụ 44. Một hình vuông có độ dài bằng 1 được chia thành 100 hình chữ nhật có
chu vi bằng nhau (hai hình chữ nhật bất kỳ không có điểm chung). Kí hiệu P là chu vi
của mỗi hình chữ nhật trong 100 hình chữ nhật này.
a) Hãy chỉ ra một cách để chia P  2,02. b) Hãy tìm giá trị lớn nhất của P.
Lời giải
a) Giả sử tồn tại cách chia hình vuông thành 100 hình chữ nhật bằng nhau có kích
thức x; y  0  x  y  thỏa mãn bài ra.

x  y  1,01 x  y  1,01
Ta có  
100xy  1 xy  0,01

Như vậy x, y là nghiệm của phương trình t 2  1,01t  0,01  0  t  0,01; t  1


Từ đó ta suy ra được x  0,01; y  1 .
Khi đó ta có cách chia như sau: Chia một cạnh hình vuông thành 100 phân bằng nhau
và bằng 0,01. Từ mỗi điểm trên cạnh đó vễ đương thẳng vuông góc với cạnh đó, cắt
dạnh đố diện tạo ra 100 hình chữ nhật bằng nhau.
b) Chú ý rằng mọi cách chia đều phải thỏa mãn điều kiện các cạnh của hình chữ nhật
phải song song với cạnh của hình vuông, nếu trái lại có một cạnh của hình chữ nhật
không song song với cạnh của hình vuông thì tất cả các hình chữ nhật khác sẽ chứa
hai cạnh đối diện song song hoặc trùng dạnh đó. Điều này là vô lí vì có những phần
của hình vuông không chia được thành hình chữ nhật.
Gọi P1 là giá trị lớn nhất của các giá trị chu vi P và  x1 ; y1  ,  x2 ; y2  , ...,  x100 ; y100 

tương ứng là kích tướng của các hình chữ nhật được chia 0  x i
 y i  với

i  1; 2; ...; 100 . Khi đó P1  2,02 .

Ta có x1  y1  x 2  y 2  ...  x100  y100  1,01 .


2 2
Mặt khác do x i  0,01; y i  1  0   y i  x i   1  0, 01 do đó 0   yi  xi   1  0, 01

Vì tổng diện tích các hình chữ nhật là 1 nên x1 y1  x2 y2  ...  x100 y100  1

33
2 2

Để ý đến ab 
a  b  a  b ta được
4
2 2 2 2 2 2

1
x 1
 y 1    x1  y1 

x 2
 y 2    x2  y 2 
 ... 
x 100
 y100    x100  y100 
4 4 4
2 2 2
1 100P1   x1  y1   x 2  y 2   x  y100  
2
1 .    ...  100
4 4  4 4 4 
 
2 2 2 2
100P12 x  y  x  y   x100  y100   100  1  0, 01
1 1 2 2
  1    ...    1
16  4 4 4  4
 

Suy ra P12  4,0804  P1  2,02 . Dấu bằng xẩy ra khi và chỉ khi
x  x 2  ...  x100  0,01
y1  x1  y 2  x 2  ...  y100  x100  0,99   1
 y1  y 2  ...  y100  1
Vậy giá trị lớn nhất của P là 2,02.
Ví dụ 45. Người ta gọi “Hình vuông (V) ngoại tiếp
tứ giác lồi ABCD” khi tứ giác ABCD nằm trong (V)
và trên mỗi cạnh của (V) có chứa đúng một đỉnh của
tứ giác ABCD (Hình 1). Giả sử tứ giác lồi ABCD có
hai hình vuông ngoại tiếp khác nhau. Chứng minh
rằng tứ giác này có vô số hình vuông ngoại tiếp nó.
Lời giải
Giả sử tứ giác lồi ABCD có hai hình N B P

vuông ngoại tiếp. Xét hình vuông MNPQ


ngoại tiếp tứ giác lồi ABCD. C

Gọi A’ là hình chiếu của A trên PQ, B’ là hình A A'

chiếu của B trên MQ. Từ B kể BE vuông góc


M Q
B' E D
với AC(E nằm trên cạnh MQ). Ta cần chứng
minh BE = AC.
- Nếu E trùng B’ thì A’ trùng C. Khi đó ta
được BE  BB'  AA'  AC
  A'
- Nếu E khác B’ ta có BB’  AA’; BBE  AC

suy ra B' BE  A ' AC nên ta được BE  AC .

34
Xét hai hình vuông M1N1P1Q1 ; M 2 N 2 P2 Q2 ngoại P2

N1 B P1
tiếp tứ giác lồi ABCD. Chứng minh tương tự N2

như trên ta được BE1  AC; BE 2  AC . Do đó


C

E1 ; E 2 trùng nhau tại D. Vậy tứ giác AC và BD A

bằng nhau và vuông goác với nhau. E1


Q2

M1 Q1
D
Ngược lại, cho tứ giác lồi ABCD có hai M
E
2
2

đường chéo bằng nhau và vuông góc với nhau.


Dựng đường thẳng d sao cho tứ giác ABCD và
d chỉ có một điểm chung là A.
Qua c dựng đường thẳng song song với d, Qua B, D dựng các đường thẳng vuông
góc với d. Các đường thẳng này cắt nhau thì được MNPQ là hình chứ nhật. Dễ dàng
chứng minh được MNPQ là hình vuông. Vậy có vô số hình vuông ngoại tiếp tứ giác
ABCD.
Ví dụ 46. Trên một đường tròn người ta sắp xếp các số 1, 2, 3, 4, 5, 6, 7, 8, 9, 10(mỗi
số xuát hiện một lần)
a) Chứng minh rằng không tồn tại cách sắp xếp mà tổng hai số kề nhau đều
lớn hơn 10.
b) Tồn tại hay không một cách sắp xếp mà tổng hai số kề nhau đề lớn hơn
hoặc bằng 10.
Lời giải
a) Gỉ sử tồn tại một cách sắp a1 1

a10 a2 9 10
xếp thỏa mãn yêu cầu bài
toán(hình 1) a9 a3
5 2

Không mất tính tổng quát ta giả a8 a4 8


6
sử a1  1 , khi đó ta có
a7 a5 4 3
a6 7
a1  a 2  10 a  9 a  10
  2  2
a1  a10  10 a10  9 a10  10 Hình 1 Hình 2

Điều nàu mâu thuẫn với giả


thiết mỗi số chỉ xuất hiện một
lần.
Vậy không tồn tại cách sắp xếp
thỏa mãn đề bài.

35
Ví dụ 47. Trên mặt phẳng cho 2011 điểm sao cho không có 3 điểm nào thẳng hàng.
Xét tất cả các đoạn thẳng nối các cặp điểm trong 2011 điểm này. Vẽ đường thẳng d
không đi qua điểm nào trong số 2011 điểm nói trên. Chứng minh rằng nếu đường
thẳng d cắt một số đoạn thẳng xét ở trên thì số đoạn thẳng bị đường thẳng d cắt là một
số chẵn.
Lời giải
Đường thẳng không đi qua điểm nào trong 2011 điểm trên nên khi d cắt một
đoạn thẳng nào đó thì nó chia mặt phẳng thành hai nửa mà 2011 điểm trên nằm ở hai
nửa mặt phẳng đối nhau và một nửa chứa chẵn số điểm, nửa còn lại chứa lẻ số
điểm(do 2011 là số lẻ). Mặt khác khi nối chẵn số điểm ở nửa bên này với lẻ số điểm
bên kia ta sẽ chứng minh được là số đoạn thẳng nối được là một số chẵn.
Thật vậy, giả sử d chia các điểm trên nửa thứ nhất có m điểm( m chẵn), và nửa
mặt phẳng kia chứa n điểm(n lẻ). Cứ một điểm bên nửa này nối được một đoạn thẳng
với nửa bên kia nên số đoạn thẳng nối được là m.n, do m chẵn nên m.n chẵn. Bài
toán được chứng minh.

Ví dụ 48. Hãy chia một tam giác bất kì thành 7 tam giác cân trong đó có 3 tam giác
bằng nhau.
Lời giải
 Trường hợp 1: Tam giác ABC không C

cân.
F E
Giả sử AB là cạnh lớn nhất của tam giác
O
ABC.
Vẽ cung tròn tâm A, bán kính AC cắt AB A G D B

tại D.
Vẽ cung tròn tâm B, bán kính BD cắt BC
tại E.
Vẽ cung tròn tâm C, bán kính CE cắt AC
tại F.
Vẽ cung tròn tâm A, bán kính AF cắt AB

36
tại G. A

Dễ dàng chứng minh 5 điểm C, D, E, F,


G thuộc đường tròn tâm O với O là tâm
I
D F
đường tròn nội tiếp tam giác ABC.
Nối 5 điểm đó với O, nối A, B với O, nối G H

F với G, D với E ta được 7 tam giác cân:


B E C
AGF,OGF,ODG, BDE,ODE,OCE,OCF .

Trong đó, có ba tam giác bằng nhau là:


OCE, OCF, OGD .

 Trường hợp 2: Tam giác ABC cân.

Giả sử tam giác ABC cân tại A. Gọi D,


E, F, G, H, I lần lượt là trung điểm các
đoạn thẳng: AB, BC, CA, DE, EF, FD.
Khi đó, ta có 7 tam giác cân ADF, BDE,
CEF, DGI, EGH, FHI, GHI trong đó ba
tam giác bằng nhau là: ADF, BDE, CEF.
Ví dụ 49. Một tứ giác lồi có độ dài bốn cạnh đều là số tự nhiên sao cho tổng ba số bất
kì trong chúng chia hết cho số còn lại. Chứng minh rằng tứ giác đó có ít nhất hai cạnh
bằng nhau.
Lời giải
Gọi độ dài các cạnh của tứ giác là a, b, c,d  a, b,c,d  N*  . Giả sử không có 2 cạnh nào

của tứ giác bằng nhau. Không mất tính tổng quát ta giả sử a  b  c  d .
Do tứ giác lồi nên a  b  c  d  a  b  c  d  3a  2a  a  b  c  d  4a
Từ giả thiết suy ra a  b  c  d chia hết cho các số a, b, c, d nên ta có a  b  c  d  3a .
Đặt a  b  c  d  mb với m   * và a  b  c  d  nc với n   *
Do a  b  c  n  m  3  n  5, m  4
Từ đó ta được 3  a  b  c  d   3a  mb  nc  3a  4b  5c

Từ đó suy ra  b – d   2  c – d   0 , điều này mâu thuẫn với a  b  c  d

Do đó tứ giác có ít nhất 2 cạnh bằng nhau.


37
Ví dụ 50. Cho đa giác đều n đỉnh. Dùng ba màu xanh, đỏ, vàng tô màu các đỉnh của
đa giác một cách tùy ý (mỗi đỉnh được tô một màu và tất cả các đỉnh đều được tô
màu). Cho phép thực hiện các thao tác sau đây: Chọn hai đỉnh kề nhau bất kì(hai đỉnh
liên tiếp) khác màu và thay màu của hai đỉnh đó bẳng màu còn lại.
a) Chứng minh rằng bằng cách thực hiện các thao tác trên một số lần ta luôn
làm cho các đỉnh của đa giác chỉ còn tô bởi hai màu.
b) Chứng minh rằng với n  4 và n  8 , bằng cách thực hiện các thao tác trên
một số lần ta luôn làm cho các đỉnh của đa giác chỉ còn tô bởi một màu.
Lời giải
a) Giả sử có đa giác đều n cạnh là A1A 2 ...An ( Ta sắp xếp các định trên một đường
tròn và các đỉnh có thứ tự theo chiều quay của kim đồng hồ). Vì dùng đúng ba màu
để tô các đỉnh nên luôn tìm được hai đỉnh kề nhau khác màu. Không mất tính tổng
quát, giả sử A1 ; A 2 là hai đỉnh kề nhau và được tô màu xanh và màu vàng. Thao tác
tô tàm 1 ta được A1 ; A 2 có màu đỏ. Ta sẽ chứng minh bằng cách thực hiện các thao
tác tô như vậy ta sẽ làm cho các đỉnh của đa giác chỉ cong được tô bởi hai màu xanh
và đỏ. Thậy vậy xét đỉnh A 3 ; A4 ; ...; A n đến đỉnh được tô màu vàng đầu tiên là
A j  3  j  n  , ta có đỉnh A j1 được tô màu xãnh hoặc màu đỏ. Nên ta thực hiện thao

tác tô lần 2, hai đỉnh A j1 ; A j sẽ có màu xanh hoặc màu đỏ. Cứ như vậy thực hiện các

thao tác tô đối với các đỉnh màu vàng tiếp theo cho đến đỉnh màu vàng cuối cùng.
Như vậy lúc này ta sẽ được một đa giác mà các đỉnh được tô bởi hai màu xanh và đỏ.
b) Với n  4 , áp dụng câu a ta được các đỉnh A1 ; A2 ; A3 ; A4 được tô bởi hai màu,
không mất tính tổng quát, giả sử hai màu đó là xanh và đỏ. Khi đó các đỉnh liên tiếp
của đa giác có thứ tự màu sắp xếp theo kiểu ĐXĐX, ĐĐXX, ĐĐĐX, ĐĐXĐ( X là
màu xanh, Đ là màu đỏ)
+ Với trường hợp ĐXĐX ta thao tác tô như sau: ĐXĐX  VVĐX  VVVV
+ Với trường hợp ĐĐXX ta thao tác tô như sau:
ĐĐXX  ĐVVX  XXVX  XĐĐX  VVĐX  VVVV
+ Với trường hợp ĐĐĐX ta thao tác tô như sau: ĐĐĐX  ĐĐVV  XXXX
+ Với trường hợp ĐĐXĐ ta thao tác tô như sau: ĐĐXĐ  ĐVVĐ
 XXĐV  XXXX

Như vậy ta đã tô được 4 đỉnh cùng một màu.


Với n  8 , ta chia 8 đỉnh thành hai bộ bốn đỉnh.
38
Áp dụng cách tô như trên ta sẽ được một bộ mà bốn đỉnh được tô cùng một màu
+ Nếu màu của hai bộ bốn đỉnh giống nhau ta được 8 đỉnh cùng màu.
+ Nếu màu của hai bộ bốn đỉnh khác nhau. Giả sử là ĐĐĐĐXXXX. Ta thực hiện
thao tác tô như sau:
ĐĐĐĐXXXX  ĐĐĐVVXXX  VVVVVVVV(theo n  4 )
Như vậy ta đã tô được 8 đỉnh cùng một màu.

Ví dụ 51. Cho hình vuông 12x12, được chia thành lưới các hình vuông đơn vị. Mỗi
đỉnh của hình vuông đơn vị này được tô bằng một trong hai màu xanh đỏ. Có tất cả
111 đỉnh màu đỏ. Hai trong số những đỉnh màu đỏ này nằm ở đỉnh hình vuông lớn,
22 đỉnh màu đỏ khác nằm trên cạnh cạnh của hình vuông lớn (không trùng với đỉnh
của hình vuông lớn ) hình vuông đơn vị được tô màu theo các quy luật sau: cạnh có
hai đầu mút màu đỏ được tô màu đỏ, cạnh có hai đầu mút màu xanh được tô màu
xanh, cạnh có một đầu mút màu đỏ và một đầu mút màu xanh thì được tô màu vàng.
Giả sứ có tất cả 66 cạnh vàng. Hỏi có bao nhiêu cạnh màu xanh.
Lời giải
Có 111 đỉnh màu đỏ, trong đó có 22 đỉnh nằm trên cạnh của hình vuông, 87 đỉnh nằm
lọt trong hình vuông lớn.Từ đó ta thấy có hai điểm màu xanh ở hai góc của hỉnh
vuông lớn, 22 điểm màu xanh trên các cạnh của hình vuông lớn không nằm trên đỉnh
của hình vuông lớn còn lại có 34 điểm màu xanh nằm lọt trong hình vuông.Với 312
cạnh của cả hình, ta cho đình của mỗi cạnh như sau: trong 2 mút của nó có i điểm
màu xanh thì cho i điểm. Gọi tổng số điểm là S, ta có S = 2(số cạnh màu xanh) + số
cạnh vàng. Ta lại có thể đếm số S theo cách khác: Mỗi điểm xanh ở góc là mút của
hai đoạn, các điểm còn lại là mút của 4 đoạn.Vậy S  2.2  22.3  34.4  206 , suy ra số
cạnh xanh là:  206 – 66  :2  70 cạnh màu xanh.

Ví dụ 52. Độ dài ba cạnh của tam giác ABC là ba số nguyên tố. Chứng minh rằng
diện tích của tam giác ABC không thể là số nguyên.
Lời giải
Giả sử a, b, c là các số nghuyên tố và là các cạnh của tam giác ABC, đặt
p abc và S là diện tích tam giác ABC. Khi đó ta có

39
16S 2  p  p  2a  p  2b  p  2c  . Giả sử S là số tự nhiên, khi đó từ hệ thức trên ta được
p  a  b  c là số chẵn, khi đó ta xét các trường hợp sau:
+ Trường hợp 1: Cả ba số a, b, c cùng chẵn, khi đó ta được a  b  c , suy ra S  3 .
+ Trường hợp 2: Trong ba số a, b, c có một số chẵn và hai số lẻ, giả sử số chẵn là a
thì ta được a  2 . Khi đó
- Nếu b  c suy ra b  c  2  a vô lí.
- Nếu b  c thì S2  b2  1   b  S  b  S   1 , hệ thức không xẩy ra vì S là số tự nhiên.
Vậy diện tích tam giác không thể là số nguyên.
Ví dụ 53. Cho bảng ô vuông kích thước 3.n (3 hàng; n cột, n là số tự nhiên lớn hơn 1)
được tạo bởi các ô vuông nhỏ kích thước 1x1. Mỗi ô vuông nhỏ được tô bởi một
trong hai màu xanh hoặc đỏ. Tìm số n bé nhất để với mỗi cách tô màu như thế luôn
tìm được hình chữ nhật tạo bởi các ô vuông nhỏ sao cho 4 ô vuông nhỏ ở 4 góc của
hình chữ nhật đó cùng màu.
Lời giải
Ta kí hiệu Đ là ô màu đỏ và X là ô màu xanh
Xét n  2; 3; 4; 5; 6 đều không thỏa mãn như hình dưới đây
Đ X Đ Đ X Đ Đ X X Đ Đ Đ X X Đ Đ Đ X X X
X Đ X X Đ X X X Đ Đ X X X Đ Đ X X X Đ Đ
X Đ X Đ Đ X Đ Đ Đ X X Đ Đ Đ X X Đ Đ X Đ

Xét n  7 . Trên một hàng có ít nhất bốn ô cùng màu. Xét bốn ô đó, giả sử bốn ô đó ở
cột thứ 1, 2, 3, 4 và được tô màu xanh. Xét trên hang 2 và 4 thuộc các cột 1, 2, 3, 4
+ Nếu có hai ô màu xanh, giả sử ô ở cột 1 và 2 thì hình vuông tạo bởi hàng 1, 2 và
cột 1, 2 thỏa mãn
1 2 3 4
1 X X X X
2 X X
3

40
+ Nếu không có hai ô màu xanh thì tồn tại ít nhất ba ô màu đỏ. Gỉ sử các ô đó thuộc
cột 1, 2, 3. Xét các hàng thuộc cột 1, 2, 3 , trong đó phải có hai ô cùng màu . Giả sử
hai ô đó thuộc cột 1, 2
- Nếu hai ô đó màu xanh thì hình chữ nhật tạo bởi hàng 1, 3 và cột 1, 2 thỏa mãn
1 2 3 4
1 X X X X
2 Đ Đ Đ
3 X X
- Nếu hai ô đó màu đỏ thì hình chữ nhật tạo bởi hàng 2, 3 và cột 1, 2 thỏa mãn
1 2 3 4
1 X X X X
2 Đ Đ Đ
3 Đ Đ
Do đó n  7.
Vậy n nhỏ nhất bằng 7.

Ví dụ 54. Mỗi ô vuông đơn vị của bảng hữu hạn m  n (m, n là các số nguyên
dương) được ghi một số thực bất kì. Xét quy tắc biến đổi sau: Mỗi lần đổi dấu tất cả
các số trên một hàng hoặc một cột. Chứng minh rằng sau một số hữu hạn bước thực
hiện quy tắc trên, ta thu được một bảng mà tổng các số trong mọi hàng và mọi cột
đều không âm.
Lời giải
Xét tổng các số trong bảng khi thực hiện phép đổi dấu. Tổng này sẽ tăng khi
tổng các số trên hàng (hoặc cột) trước khi đổi dấu là âm; tổng không đổi khi khi tổng
các số trên hàng (hoặc cột) trước khi đổi dấu bằng 0 và tổng này sẽ giảm khi tổng các
số trên hàng (hoặc cột) trước khi đổi dấu là dương.
Khi thực hiện phép biến đổi trên thì mỗi số trong một ô hoặc là trùng với số
ban đầu hoặc là số đối của số ban đầu, vì thế khi thực hiện phép biến đổi trên thì có
thể nhận được tối đa 2 m.n bảng khác nhau, do vậy tổng tất cả các số trong bảng chỉ
nhận hữu hạn giá trị khác nhau.

41
Ta xét bảng ban đầu: Nếu không có hàng nào và không có cột nào có tổng các
số là âm thì bài toán được chứng minh.
Ngược lại, ta chọn một hàng hoặc cột có tổng các số là âm, ta thực hiện phép
biến đổi trên hàng hoặc cột đó. Khi đó trong bảng vừa nhận được nếu có hàng hoặc
cột có tổng các số là âm thì ta lại thực hiện phép biến đổi trên hàng hoặc cột đó và
nhận được bảng mới và cứ tiếp tục như vậy. Như vậy mỗi lần thực hiện phép biến đổi
thì tổng các số trong bảng tăng lên.
Do tổng đó chỉ nhận hữu hạn giá trị nên hoặc là tại một bước thực hiện nào đó
ta nhận được bảng cần tìm, hoặc là nhận được bảng có tổng đạt giá trị lớn nhất.
Bảng có tổng đạt lớn nhất thỏa mãn bài toán vì nếu trong nó có một vệt có tổng
là âm thì thực hiện phép biến đổi trên vệt đó ta được bảng mới có tổng tăng lên lớn
hơn tổng lớn nhất đó, vô lý.
Vậy trong mọi trường hợp ta đều có điều phải chứng minh.
Ví dụ 55. Xét n điểm điểm liên tiếp A1 ; A 2 ; A3 ;...; An trên cùng một đường thẳng thỏa
mãn điều kiện
A1 A 2  A 2 A 3  A 3 A 4  ...  A n 1A n
Tìm n biết rằng trên đường thẳng đó có đúng 2025 đoạn thẳng nhân một trong các
điểm trên làm trung điểm.
Lời giải
Giả sử n điểm A1 ; A 2 ; A 3 ;...; A n thỏa mãn A1A2  A2 A3  A 3 A 4  ...  A n1A n được biểu
diễn bằng hình vẽ sau

A1 A2 A3 Ai An+1-i An-2 An-1 An

Khi đó ta có
+ Các điểm A1 và A n không là trung điểm của đoạn thẳng nào
+ Các điểm A 2 và A n 1 là trung điểm của một đoạn thẳng.
+ Các điểm A 3 và A n 2 là trung điểm của hai đoạn thẳng.
... ... ...
+ Các điểm Ai và A n i 1 là trung điểm của i  1 đoạn thẳng.
Với mọi giá trị i  2; 3; 4;... trong đó 2i  n .

42
Đến đây ta xét các trường hợp sau
 Trường hợp 1: Nếu n là số lẻ, khi đó ta đặt n  2k  1 k  N* . Ta có  
k  k  1  n  k  1 . Từ đó sẽ có một điểm A k 1 là trung điểm của k đoạn thẳng.

Từ đó ta suy ra số các đoạn thẳng nhận các điểm đã cho làm trung điểm là
S  1  2  3  ...   k  1  k   k  1  ...  3  2  1   k  1 k  k  k 2 .

Theo bài ra thì có 2025 đoạn thẳng nhân một trong các điểm trên làm trung điểm nên
ta được k 2  2025 , do đó ta được k  45 nên suy ra n  91 .

 
 Trường hợp 2: Nếu n là số lẻ, khi đó ta đặt n  2k k  N* . Ta có k  k  1  n  k  1 .

Từ đó ta thấy trong các điểm đã cho không có điểm chính giữa.


Từ đó ta suy ra số các đoạn thẳng nhận các điểm đã cho làm trung điểm là
S  1  2  3  ...   k  1   k  1  ...  3  2  1   k  1 k .

Theo bài ra thì có 2025 đoạn thẳng nhân một trong các điểm trên làm trung điểm nên
k  k  1  2025 , Dễ thấy không tồn tại k thỏa mãn đẳng thức trên. Do đó không có n

điểm thỏa mãn bài toán.


Vậy n  91 thỏa mãn yêu cầu bài toán.
Ví dụ 56. Cho hai tam giác đều bằng nhau, một tam giác có cạnh màu đỏ, một tam
giác có cạnh màu xanh. Hai tam giác này nằm chồng lên nhau sao cho phần chung là
một lục giác mà kí hiệu là MNPQRS. Chứng minh rằng lục giác MNPQRS có tổng
độ dài các cạnh màu đỏ bằng tổng độ dài các cạnh màu xanh.
Lời giải
Giả sử hai tam giác đều ABC và C1
A

A1 B1C1 bằng nhau và nằm chồng lên nhau R

sao cho lục giác MNPQRS được kí hiệu như S

Q
hình vẽ.
B
Các góc của hai tam giác đều ABC M

và A1 B1C1 bằng nhau và hai góc đối đỉnh thì A1 N P B1

C
bằng nhau nên ta nhận thấy các tam giác
A1MN; B1PQ; C1RS; CPN; ARQ; BMS đồng

43
dạng với nhau. Từ đó ta được
MN PQ RS NP QR SM
    
A1M  A1N B1P  B1Q C1R  C1S CN  CP AQ  AR BS  BM

Theo tính chất dãy tỉ số bằng nhau ta được


MN  PQ  RQ NP  QR  MS

A1M  A1N  B1P  B1Q  C1R  C1S CN  CP  AQ  AR  BM  BS

Đặt độ dài cạnh của tam giác đều là a và x  MN  PQ  RQ; y  NP  QR  MS .


x y
Khi đó ta được   x  3a  y   y  3a  x    x  y  3a  x  y   0
3a  y 3a  x

Mặt khác ta có
3a  BC  CA  AB   BM  MN  NC    CP  PQ  QA    AR  RS  SB 
  CN  CP    AQ  AR    BS  BM    MN  PQ  RS 
  NP  QR  MS    MN  PQ  RS   y  x

Hay ta được 3a  x  y  3a  x  y  0
Do đó từ đẳng thức trên ta suy ra được x  y hay MN  PQ  RS  NP  QR  SM .

Ví dụ 57. Cho đa giác A1A 2 A 3 A 4 A 3 A6 A7 A8 có các cạnh là các số hữu tỉ và các góc
của đa giác bằng nhau. Chứng minh rằng các cặp cạnh đối diện song song và bằng
nhau.
Lời giải
Do đa giác có các góc ở các đỉnh bằng nhau A1 A2
A B
nên mỗi góc có số đo là
A8 A3
 n  2  .180 0


6.1800
 1350 .
n 8 A7 A4

Kéo dài các cạnh của đa giác, chúng cắt nhau


D A6 A5 C
tại các điểm A, B, C, D như hình vẽ.

Khi đó ta có AA A  450 , hoàn toán tương tự
1 8

ta được các góc ngoài của đa giác đều có số


đo là 450 .

44
Nên các tam giác AA1A 8 , BA 2 A 3 , CA 4 A 5 , DA6 A7 là các tam giác vuông cân. Từ đó ta
được tứ giác ABCD là hình chữ nhật. Suy ra các cạnh đối diện của đa giác
A1A 2 A 3 A 4 A 3 A 6 A7 A8 song song với nhau.

A1A 8
Xét tam giác AA1A8 ta có AA1  .
2

A 2 A3 A4 A5 A 6 A7
Tương tự ta có BA2  ; CA 5  ; DA6 
2 2 2

A1A 8 A2 A3
Khi đó ta được AB  AA1  A1A 2  A 2 B   A1 A 2 
2 2

A4 A5 A 6 A7
Tương tự ta cũng có CD   A 5 A6 
2 2

A1 A 8 A2 A 3 A4 A 5 A 6 A7
Do AB  CD nên ta được  A1 A 2    A 5 A6 
2 2 2 2

A1 A 8 A2 A3 A4 A5 A 6 A7 1
Suy ra A1A 2  A 5 A6      A A
1 8
 A 2 A 3  A 4 A 5  A 6 A7 
2 2 2 2 2

Do các cạnh của đa giác A1A 2 A 3 A4 A3 A6 A7 A8 là các số hữu tỉ nên A1A 2  A5 A6 là số


hữu tỉ
1
Suy ra A A1 8
 A 2 A 3  A 4 A 5  A6 A7  là số hữu tỉ.
2

1
Do đó A A1 8
 A 2 A 3  A 4 A 5  A 6 A7   0 , nên ta được A1A 2  A 5 A6  0 hay
2

A1A 2  A 5 A 6

Hoàn toàn tương tự ta chứng minh được A 2 A 3  A6 A7 ; A 3 A4  A7 A8 ; A4 A5  A8 A1


Vậy bài toán được chứng minh xong.
Ví dụ 58. Cho ngũ giác ABCDE, mà mỗi đường chéo song song với một cạnh của
ngũ giác. Chứng minh rằng tỉ số giữa cạnh của ngũ giác và đường chéo song song với
nó là một hằng số.
Lời giải

45
Gọi O là giao điểm của CE và BD. Qua C kẻ A

đường thẳng song song với BD cắt AB tại P, từ


đó ta thấy O nằm trong ngũ giác và P nằm ngoài B
E
ngũ giác ABCDE.
P
OC O
Đặt  k  k  0  . Do AE//BD, AB//CE và
AM D C

BE//CD nên ta được OBE  AEB và


OCD ∽ AEB
Do đó ta được OCD ∽ ABE suy ra
OC OD
  k  OD  k.AE
AB AE
Dễ thấy các tứ giác ABOE, APCE và BPCO là các hình bình hành, nên ta được
AB  OE; CP  AE và BP  OD  k.AB .

  ACE
Xét hai tam giác EOD và APC có PAC   CED
 và EOD
  OEA
  APC

OD OE AE.k AB AB
Do đó EOD ∽ APC suy ra    
PC PA AE AB  BP AB  k.AB

1 5 1
Từ đó ta được k   k2  k  1  0  k 
1 k 2

EC OE  OC AB  OC 5 1
Do đó ta có    1 k 
AB AB AB 2

AC AD BE BD 5 1
Tương tự ta tính được    
DE BC CD AE 2

Ví dụ 59. Tính diện tích của ngũ giác lồi ABCDE, biết các tam giác ABC, BCD,
CDE, DEA, EAB cùng có diện tích bằng 1.
Lời giải
Gọi I  EC  BD. Ta có S BAE  S DAE nên A B

khoảng cách từ B, D đến AE bằng nhau.


Do B, D cùng phía đối với đường thẳng
AE nên BD//AE. Tương tự ta được E I C

AB//CE
D
Do đó ABIE là hình bình hành nên

46
S IBE  S ABE  1

Đặt S ICD  x  0  x  1

 S IBC  S BCD  S ICD  1  x  S ECD  S ICD  S IED

 3 5
 x
S ICD IC S IBC x 1 x 2
Lại có   hay   x 2  3x  1  0  
S IDE IE S IBE 1 x 1  3 5
x 
 2

3 5 5 1
Kết hợp điều kiện ta có x   SIED 
2 2

5 1 5  5
Do đó S ABCDE  S EAB  S EBI  S BCD  S IED  3   .
2 2

Ví dụ 60. Cho hình bình hành ABCD và một điểm E nằm trên cạnh CD. Biết rằng
.
các tam giác BCE, BED và DAB là tam giác cân. Tính số đo của góc BAD
Lời giải
  BCD
Đặt BAD    ; ABD
  BDC
   ; ADB
   ; EBC
   ; BED
 

Nếu   900 , khi đó chỉ có B C


β α
δ
AB  AD, BC  CE , mâu thuẫn với giả
γ E
thiết E nằm trên đoạn CD.
Do đó   900 , khi đó ta xét các trường
α φ δ
hợp sau A D

+ Trường hợp 1: Nếu BC  CE , khi đó ta


được
    900  
CBE
2

    180 0    90 0    90 0
Lại có BED
2
Do đó ta được EB  ED nên tam giác BED cân tai E
    1 180 0    1 180 0  
  EBD
Suy ra EDB   
2
 2
3
Do đó ta được  
4

1800   

47
1800   360
- Nếu    thì ta được    360 hay BAD
5

3.1800   3.180
0
- Nếu    thì ta được   hay BAD
7 7

+ Trường hợp 2: Nếu BC  BE , khi đó ta được   1800    900 do đó EB  ED


0
    180   và       
Suy ra tam giác BDE cân tại E nên ta được BDE
2
Ta có tam giác ABD cân và    nên ta được   
5 2.1800   720
Do đó 1800             720 hay BAD
2 5
+ Trường hợp 3: Nếu EC  EB , khi đó ta được    ,   2
- Nếu     2 ,      ta được     2
1800   360
Do đó ta được 1800        5     360 hay BAD
5

180 0  
- Nếu    900   suy ra           900
2
  450
Do đó     450 hạy BAD
- Nếu   2 thì   3 do đó ta được    hoặc   
1800
Do đó ta được   360 hoặc 3  1800  4   
7
0
  180
  36 0 hoặc BAD
Hay BAD
7
0 0
     180 ; 360 ; 450 ; 720 ; 3.180 
Vậy ta được BAD  
 7 7 

Ví dụ 61. Cho ngũ giác lồi ABCDE có diện tích S. Chứng minh rằng trong năm tam
S
giác ABC, BCD, CDE, DEA, EAB luôn có một tam giác có diện tích lớn hơn .
4

Lời giải

48
Chú ý rằng nếu điểm K nằm trên cạnh PQ B C
của tứ giác lồi MNPQ thì ta luôn có
K
Min S PMN ; S QMN   S KMN  Max S PMN ; S QMN 
S D
A
+ Nếu S ABC  thì tam giác ABC là tam giác
4
cần tìm.
S
+ Nếu S BCD  thì tam giác BCD là tam giác
4
E
cần tìm.
S S
+ Nếu S ABC  và S BCD  thì ta được
4 4
S
S ABCDK  S ABC  S BCD  S KBC 
2
S
Như vậy một trong hai tam giác KDE hoặc KEA có diện tích lớn hơn
4
S
+ Nếu S KDE  thì theo chú ý trên trong tứ giác ACDE có điểm K nằm trên cạnh AC.
4
S S
Khi đó ta được S CDE  S KDE  hoặc S DEA  S KDE 
4 4
S S S
+ Nếu S KEA  thì tương tự ta có S DEA  hoặc S DEA 
4 4 4
S
Vậy trong năm tam giác trên luôn có một tam giác có diện tích lớn hơn .
4

Ví dụ 62. Cho lục giác lồi ABCDEF có diện tích S và AB song song với DE, BC
song song với EF, CD song song với FA.
S
a) Chứng minh rằng S ACE  b) Chứng minh rằng S ACE  S BDF
2
Lời giải
a) Qua A, C, E kẻ các đường thẳng a, c, e lần B C

lượt song song với BC, AB, AF. Giả sử a cắt c


tại M, c cắt e tại N, e cắt a tại P. Khi đó dễ A
P
M D

thấy các tứ giác ABCM, CDEN, EFAP là hình N

bình hành. Các hình bình hành này đôi một


nằm ngoài nhau và cùng nằm trong đa giác F E

ABCDEF. Từ đó ta được

49
1 S
S ABC  S CDE  S EFA 
2
 S ABCM  S CDEN  S EFAP  
2
S
Suy ra S ACE  .
2

Dễ thấy MN  AB  DE ; NP  CD  AF ; PM  BC  EF

S S
Từ đó ta thấy S ACE   S ABC  S CDE  S EFA   S ABCM  S CDEN  S EFAP  S
2 2
Hay S MNP  0  ba điểm M, N, P trùng nhau  AB  DE; CD  AF; BC  EF
b) Hoàn toàn tương tự như trên qua B, D, F C D

kẻ các đường thẳng b, d, f lần lượt song song


P'
với AF, BC, AB. Giả sử d cắt f tại M’, f cắt b B N' E

M'
tại N’, b cắt d tại P’. Khi đó ta có
M' N'  AB  DE  MN
A F
N'P '  CD  AF  NP
P'M '  BC  EF  PM

Do đó ta được MNP  M ' N' P ' . Từ đó suy


ra
S ABCM  S CDEN  SEFAP  S  S MNP  S  S M' N'P'  S DEFM'  S FABN'  S BCDP'

Từ đó ta có S ABC  S CDE  SEFA  S DEF  SFAB  S BCD nên suy ra S ACE  S BDF

Ví dụ 63. Cho tam giác ABC chứa hình bình hành MNPQ. Chứng minh rằng
1
S MNPQ  S
2 ABC

Lời giải
Ta xét các trường hợp sau đây:
+ Trường hợp 1: Tứ giác MNPQ có một cạnh A

nằm trên cạnh của tam giác ABC. Không mất


tính tổng quát ta giả sử cạnh MN nằm trên cạnh
BC và M nằm giữa hai điểm B, N. F
Q P
E

Gọi giao điểm của PQ với AB, AC lần lượt là F


và E. Qua E dựng đường thẳng song song với
B M D N C

50
AB cắt BC tại D, khi đó tứ giác BDEF là hình
bình hành và có cùng đường cao với hình bình
hành MNPQ.
Mà ta lại có PQ  EF nên ta được S MNPQ  S BDEF
1 1
Lại dễ thấy S BDEF  S ABC nên ta được S MNPQ  S ABC
2 2
+ Trường hợp 2: Tứ giác MNPQ không có A

cạnh nào nằm trên cạnh của tam giác ABC.


Khi đó không mất tính tổng quát ta giả sử giao Q' P'

P
điểm của MQ và NP với các cạnh của tam giác Q

Q1 P1
ABC được kí hiệu như hình vẽ.
M N
Dễ thấy tứ giác M’N’P’Q’ là hình thang có hai
B M' N' C
đỉnh nằm trên một cạnh của tam giác ABC.
Hai điểm M’ và Q hoặc hai điểm N’ và P’
không thể cùng thuộc một cạnh của tam giác
ABC vì MQ và NP không nằm tren một cạnh
của tam giác ABC. Hai đỉnh M’ và P’ hoặc hai
đỉnh N’ và Q’ cũng không thể cùng nằm trên
một cạnh của tam giác ABC.
Như vậy hai đỉnh M’ và N’ hoặc P’ và Q’ cùng nằm trêm một cạnh của tam giác
ABC. Giả sử M’N’ nằm trên cạnh BC. Khi đó trên đoạn N’P’ và M’Q’ lấn lần lượt
các điểm P1 ; Q1 sao cho N' P1  NP; M 'Q1  MQ . Khi đó ta được hình bình hành
M' N'P1Q1 có cạnh M’N’ nằm trên cạnh BC và tam giác ABCD chứ các điểm M’, N’,

P’, Q’. Do đó chứng minh tương tự như trên ta được


1 1
S M' N'P1Q1  S ABC  S MNPQ  S ABC .
2 2

Ví dụ 64. Cho đa giác lồi (H) có n cạnh. Kẻ các đoạn thẳng nối trung điểm của hai
cạnh kề nhau của hình (H) tạo thành đa giác (H’).
1
a) Chứng minh rằng nếu n  4 thì SH'  S H
2
1
b) Chứng minh rằng nếu n  5 thì SH'  S H
2
51
SH'  1 1
c) Lấy ví dụ về ngũ giác lồi (H) thỏa mãn điều kiện   6
SH  2 10

Lời giải
a) Xét đa giác lồi ABCD như hình vẽ. Gọi M, N, A2 B2 A3

P, Q lần lượt là trung điểm của AB, BC, CD, M2


B1 B3
DA. Khi đó MQ là đường trung bình của tam M1
A1 A4
1
giác ABD nên ta được S AMQ  S ABD B4
4
A5
An
Tương tự ta có
1 1 1
S MBN  S ABC ; S NCP  S BCD ; S PDQ  S ACD
2 2 2
Từ đó ta được
S AMQ  S MBN  S NCP  S PDQ
1 1 1 1 1
 S ABD  S ABC  S BCD  S ACD  S ABCD
4 2 2 2 2
1
Do đó suy ra S MNPQ  S ABCD .
2
b) Giả sử n-giác lồi là A1A 2 A 3 ...A n và trung điểm các cạnh A1A 2 ; A 2 A 3 ; ...; A n A1 lần
lượt là B1 ; B2 ; B3 ;...; Bn . Khi đó ta có đa giác H là A1A 2 A 3 ...A n và đa giác H’ là
B1 B2 B3 ...Bn .

Các phân của đa giác (H) nằm ngoài đa giác (H’) là các tam giác
B1A 2 B2 ; B2 A 3 B3 ; B3 A 4 B4 ;...; Bn A1 B1

1
Ta có S B A B  S B A B  S B A B  ...  S B A B 
1 2 2 2 3 3 3 4 4 n 1 1

S
4 A1 A 2 A 3
 S A2 A3 A4  S A3 A4 A5  ...  S An A1A2 
Trong n tam giác A1A 2 A 3 ; A 2 A 3 A 4 ; ...; A n A1A 2 thì hai tam giác liên tiếp mới có phần
chung và hai tam giác không liên tiếp không có điểm chung trong. Ngoài ra mỗi tam
giác trên chứa một tam giác không có điểm chung trong với các tam giác khác, chẳng
hạn tam giác A1M1M 2 nằm trong tam giác A1A 2 A 3 và không có điểm chung trong
với các tam giác A 2 A 3 A 4 ; A 3 A 4 A 5 ; ...; A n A1A2 .
Do đó ta được S A A A  S A A A  ...  S A A A  2S H
1 2 3 2 3 4 n 1 2

52
1 1
Suy ra S B A B  S B A B  S B A B  ...  S B A B 
1 2 2 2 3 3 3 4 4 n 1 1
4

S A1A2 A3  S A2 A3A4  S A3 A4 A5  ...  S An A1A2  SH
2

1
Hay ta được SH'  S H .
2
c) Xét tam giác đều có cạnh bằng a và I là trung điểm của BE. Kẻ hai đoạn thẳng IC,
ID có độ dài bằng 1 lần lượt song song và cùng cùng chiều với AB, AE. Khi đó tam
giác ICD là tam giác đều và ta được ngũ giác lồi (H) là ABCDE.
Gọi M, N, P, Q, R lần lượt là trung điểm của AB, A

BC, CD, DE, EA. Khi đó ngũ giác lồi H’ là


MNPQE. Từ đó ta tính được M R

1 a2 . 3 1 1 a2 3
S AMR  S ABE  và S BMN  S ABC  S ABI  B
I
4 16 4 4 32 E

N Q
a2 3
Hoàn toàn tương tự ta được S EQR  . C P D
32
Đặt S n  S AMR  S BMN  S CNP  S DPQ  S EQR thì ta được

a2 3
S n  S AMR  S BMN  S EQR 
8

a 2 3 BE  CE a2 3 a  1
Ta có SH  S ABCDE  S ABE  S BCDE   .IP  
4 2 4 2

a2 3 a  1 a2 3 a2 3 a  1
Do đó ta được SH'  S H  S n     
4 2 8 8 2

a2 3 a  1
S H' 
a2 3 8 2  1  2. a  1
Do SH  S ABE  nên 
4 S H  2
a 3 2 a2 3
4

4.106 2  a  1 4a 4 4 1
Do đó với a  thì    
3 a 2
3 a 2
3 a 3 4.10 6 10 6
. 3
3

S H' 1 1 4.106
Suy ra   6 . Vậy ngũ giác ABCDE chọn như trên và a  thì ta được
SH  2 10 3

S H' 1 1
  6 .
SH  2 10

53
Ví dụ 65. Cho đa giác lồi (H) có diện tích S. Chứng minh rằng luôn tồn tại một hình
tam giác có diện tích không vượt qua 2S chứa hình (H).
Lời giải
Trong các tam giác có ba đỉnh là ba đỉnh của hình (H) ta chọn tam giác có diện tích
lớn nhất. Gọi ba đỉnh của tam giác đó là A, B, C. Khi đó ta xét các trường hợp sau:
S
+ Trường hợp 1: Nếu S ABC  C'
2
Qua A, B, C kẻ các đường thẳng lần lượt song A
B
song với BC, CA, AB tạo thành tam giác
E
A’B’C’. Dễ thấy tam giác A’B’C’ chứa hình
B' C A'
(H) vì nếu một đỉnh của hình (H) nằm ngoài
tam giác A’B’C’, chẳng hạn điểm E như hình
vẽ thì khi đó S EBC  S ABC , điều này trái với cách
chọn tam giác ABC.
Mặt khác dễ thấy A, B, C lần lượt là trung điểm của B’C’, C’A’, A’B’.
Từ đó ta được SA' B'C'  4S ABC  2S .
S A'
+ Trường hợp 2: Nếu S ABC 
2
Trong các đỉnh của hình (H)nằm khác phía với A A

so với BC ta chọn đỉnh xa nhất, không mất tính N


P
tổng quát ta giả sử điểm đó là M. Tương tự gọi
các đỉnh khác phía B, C đối với CA, AB và xa B C

CA, AB nhất lần lượt là N, P. Khi đó hình (H) B' M C'

chứa lục giác APMBCN.


S
Từ S ABC  ta được S APB  S BMC  SCNA  S ABC .
2
Qua M, N, P kẻ các đường thẳng lần lượt song song với BC, CA, AB và các đường
thẳng này cắt nhau tạo thành tam giác A’B’C’. Theo cánh chọn M, N, P thì tam giác
A’B’C’ chứa hình (H). Theo kết qua bài toán trên ta có SA' B'C'  2SAPBMCN  2S .
Nếu BC là một cạnh của hình (H) tức là không có đỉnh nào của hình (H) nằm khác
phía đối với A so với BC. Qua N, P kẻ hai đường thẳng lần lượt song song với CA,
54
AB. Hai đường thẳng đó tạo với đường thẳng BC tam giác A’B’C’. Khi đó dễ thấy
tam giác A’B’C’ chứa hình (H) và SA' B'C'  2SAPBNC  2S .
Nếu tam giác ABC có hai cạnh chẳng hạn BC, CA là hai cạnh của hình (H) thì qua P
kẻ đường thẳng song song với AB, đường thẳng này tạo với hai đường thẳng BC, CA
tam giác A’B’C’. Khi đó tam giác A’B’C’ chứa hình (H) và SA' B'C'  2SAPBC  2S

Ví dụ 66. Cho đa giác lồi (H) có diện tích S. Chứng minh rằng (H) chứa một hình
tam giác có diện tích:
S 3
a) Lớn hớn b) Không bé hơn S
4 8

Lời giải
Giả sử hai đỉnh xa nhất của hình (H) là A, B. Qua A, B kẻ hai đường thẳng a, b kẻ hai
đường thẳng vuông góc với AB. Khi đó dễ thấy hình (H) nằm giữa hai đường thẳng a
và b.
A
M N
Q A M
I d1
C D

Q H d2 P
R T
F E
K d3
F E

V U
P B N B

a) Đường thẳng AB chia hình (H) thành hai đa giác  H1  và  H2  nằm về hai phía

của đường thẳng AB. Gọi hai đỉnh của  H1  và  H 2  xa AB nhất lần lượt là E, F thì

các tam giác ABE, ABF nằm trong hình (H), Qua E, F kẻ các đường thẳng song song
với AB, chúng tạo với a, b một hình chữ nhật MNPQ chứa hình (H) và không trùng
với hình (H).
1 1 S S
Dễ thấy S ABE  S ABF  S MNPQ  S nên ta được S ABE  hoặc S ABF  .
2 2 4 4
b) Chia AB thành bốn đoạn thẳng bằng nhau AI  IH  HK  KB  x . Qua I, K, H kẻ ba
đường thẳng d1 ,d 2 ,d 3 vuông góc với AB và đường thẳng d 2 chia hình (H) thành hai
đa giác  H1  và  H2  .

55
Giả sử d1 cắt biên của hình  H1  tại C, D. Kéo dài các cạnh chứa C, D cắt a, d2 tạo

thành hình thang MNPQ chứa hình  H1  . Giả sử d 3 cắt biên của hình  H2  tại E, F.

Kéo dài các cạnh chứa E, F cắt d 2 , b tạo thành hình thang RTUV chứa hình  H2  .

Vì hình thang MNPQ và RTUV chứa lần lượt các đa giác  H1  và  H2  nên ta được

S  S H   S H   S MNPQ  S RTUV  CD.2x  EF.2x  2x  CD  EF 


1 2

3x 3x 3x 3
S AEF  S BCD  EF.
2
 CD.
2

2
 EF  CD   .2x  CD  EF 
4
3 3 3
Từ đó ta được S AEF  S BCD  S nên suy ra S AEF  S hoặc S BCD  S
4 8 8
Ngoài ra do hình (H) là đa giác lồi nên hình (H) chứa các tam giác AEF và BCD.

Ví dụ 67. a) Chứng minh rằng nếu tam giác ABC có ba đường phân giác có độ dài
1
không vượt quá 1 thì ta luôn có S ABC 
3
b) Chứng minh rằng nếu tam giác ABC có ba đường phân giác có độ dài
1
không nhỏ hơn 1 thì ta luôn có S ABC 
3
Lời giải
Giả sử ba đường phân giác AD, BE, CF của A

tam giác ABC cắt nhau tại I.


a) Giả sử AD  1; BE  1; CF  1 và BC là E
F

cạnh lớn nhất của tam giác ABC. I

Khi đó ta có AF  BF; AE  CE nên


B D C
1
S AEF  S
4 ABC
3
Do đó ta được S BCEF  S ABC hay
4
4
S ABC  S
3 BCEF
 
  B  C  600
  600 nên ta được BIF
Do A
2

56
4 4 1  sin 600  . 2 2 3 1
Từ đó suy ra S ABC  S ABC  . BE.CF.sin BIF 
3 3 2 3 3 2 3

b) Giả sử AD  1; BE  1; CF  1 và AC là cạnh bé nhất của ta giác ABC.


1
Khi đó ta có BF  AF; BD  CD nên ta được S BDF  S ABC
4
3 4
Do đó suy ra S ACDF  S ABC hay S ABC  S ACDF .
4 3

Do AC là cạnh nhỏ nhất của tam giác ABC nên B  60 0 nên A


 C
  120 0

 
  A  C  600 nên ta được 600  CID
Ta có CID   900 .
2

4 4 1   sin 60 0  . 2 2 3 1
Do đó S ABC  S ACDF  . .AD.CF.sin CID 
3 3 2 3 3 2 3

Ví dụ 68. Cho đa giác lồi (H) có diện tích S và chu vi 2p. Chứng minh rằng:
S
a) Nếu (H) chứa đường tròn (O; r) thì r  .
p
S
b) Nếu (H) luôn dựng được đường tròn có bán kính r  .
p
Lời giải
a) Xét đa giác lồi A1A 2 A 3 ...A n , do đa giác chứa đường tròn (I; r) nên các khoảng cách
từ I đến các cạnh của đa giác đều không nhỏ hơn r. Khi đó ta được
1
S  S IA1A2  S IA2 A3  ...  S IAn A1   A A .d  A2 A3 .d2  ...  An A1 .dn 
2 1 2 1
1
  A A  A2 A3  ...  An A1  r  p.r
2 1 2
Trong đó d1 ; d 2 ; ...; d n là khoảng cánh từ I đến A1A 2 ; A 2 A 3 ; ...; A n A1 và p là nửa chu vi
của đa giác.
S
Từ đó ta suy ra được r  .
p

57
b) Ứng với mỗi cạnh A i Ai 1 dựng một hình chữ nhật Ai Ai1M i Ni nằm trong đa giác
S
A1A 2 A 3 ...A n sao cho A1Ni  . Ta gọi hình chữ nhật Ai Ai1M i Ni là  Ci  . Khi đó ta
2p

A i A i 1
có SC   .S
1
2p

Từ đó ta được
AA AA AA A A
S C   S C   S C   ...  S C   1 2 .S  2 3 .S  3 4 .S  ...  n 1 .S
1 2 3 n
2p 2p 2p 2p
A A  A 2 A 3  ...  A n A1
 S. 1 2 S
2p

Gọi  H0  hình bằng hợp của n hình chữ nhật nói trên, khi đó do hai hình chữ nhật

liên tiếp có phân hồng lên nhau nên ta được SH   S C   SC   S C   ...  S C   S . Điều
0 1 2 3 n

này có nghĩa là hình  H0  không phủ kín được đa giác A1A 2 A 3 ...A n . Điều này chứng

tỏ tồn tại một điểm O nằm ngoài tất cả các hình chữ nhật  C1  ;  C2  ; ...;  Cn  .

Giả sử trong các đường thẳng A1A 2 ; A 2 A 3 ;...; A n A1 thì Ai Ai1 là đường thẳng gần O
S
nhất và gọi d là khoảng cách từ O đến Ai Ai 1 . Ta sẽ chứng minh d  .
2p

Thật vậy, qua điểm O kẻ đường thẳng vuông góc với A i Ai 1 và M i Ni lần lượt tại L và
K. vì đoạn Mi Ni và điểm O nằm về một phía so với Ai Ai 1 và điểm O nằm ngoài hình
S
chữ nhật A i A i1M i Ni nên ta được d  OL  KL  A i Ni  .
2p

Như vậy đường tròn tâm O có bán kính r  d nằm trong đa giác A1A 2 A 3 ...A n .

Ví dụ 69. Bên trong đường tròn tâm O bán kính 1 cho tam giác ABC có diện tích lớn
hơn hoặc bằng 1. Chứng minh rằng điểm O nằm trong hoặc nằm trên cạnh của tam
giác ABC.
Lời giải

58
Giả sử O nằm ngoài miền tam giác ABC. Không mất tính A

tổng quát, giả sử A và O nằm về 2 phía của đường thẳng


K
BC. Suy ra đoạn AO cắt đường thẳng BC tại K. Kẻ AH B H
O
C

vuông góc với BC tại H. Suy ra AH  AK  AO  1 suy ra


AH < 1
AH.BC 2.1
Suy ra S ABC    1 (mâu thuẫn với giả thiết).
2 2
Suy ra điều phải chứng minh.
Ví dụ 70. Chứng minh rằng nếu ba điểm A, B, C không có điểm nào nằm bên ngoài
đường tròn (O) sao cho tam giác ABC có ba góc nhọn thì chu vi của đường tròn
ngoại tiếp ABC không lớn hơn chu vi (O).
Lời giải
Gọi đường tròn ngoại tiếp tam giác ABC là (I), I nằm trong tam giác ABC.
- Nếu A, B, C nằm trên đường tròn (O) thì hai đường tròn (I) và (O) trùng nhau.
- Nếu (O) đựng (I) hoặc (O) tiếp xúc trong với (I) thì chu vi của (I) nhỏ hơn chu vi
của (O).
- Nếu (O) và (I) cắt nhau tại M và N. Vì tam giác ABC nhọn nên số đo cung nhỏ MN
nhỏ hơn 1800 và số đo cung lớn MN lớn hơn 1800 , do đó tồn tại đường kính của (I)
nằm trong (O). Do đó chu vi của (I) nhỏ hơn chu vi của (O).

Bài 2. Cho 9 hình vuông có các độ dài cạnh tính bằng mét là
n  1, n  2, n  3, n  4, n  5,

n  6, n  7, n  8, n  9 , với n là số nguyên dương. Gọi S là tổng diện tích của 9 hình

vuông này.
Có hay không một hình vuông diện tích bằng S và có độ dài cạnh là một số
nguyên mét?
b) Giả sử tồn tại hình vuông diện tích bằng S và có độ dài cạnh là a  a  N *  . Ta có:
2 2 2 2 2 2 2 2 2
 n  1   n  2    n  3    n  4    n  5    n  6    n  7    n  8    n  9   a 2 Hay

9n 2  90n  285  a 2 . Chú ý 9n 2  90n  285 là số nguyên chia cho 9 dư 6.

Số chính phương a 2 chia cho 9 chỉ có thể dư 0; 1; 4; 7 . Ta gặp mâu thuẫn.


59
Vậy không tồn tại hình vuông thỏa bài toán.

60

You might also like